Download as doc, pdf, or txt
Download as doc, pdf, or txt
You are on page 1of 50

1 / 50

1. Which of the following statements regarding the second heart sound is TRUE?
A. It is preceeded by a long internal
B. It is closely related to the closure of the S-L valve
C. Its duration is relatively long (0.14 sec.)
D. It is soft in character
Reference: Textbook of Medical Physiology, 10th ed., by Guyton and Hall
pp. 245 MPL 0.50

2. One of the following does not contribute to the production of the first heart sound
A. Vibration of the semilunar valves immediately after closure
B. Vibration of the walls of the heart
C. Vibration of the adjacent column of blood
D. Vibration of the major vessels around the heart
Reference: pp. 245 MPL 0.75

3. Renal autoregulation of blood flow is best explained by:


A. Oxygen demand theory C. Myogenic theory
B. Neurogenic theory D. Metabolic theory
Reference: pp. 178 MPL 0.50

4. The tendency for turbulent flow rises in direct proportional to:


A. Velocity of flow C. Stroke volume
B. Hematocrit D. Cardiac output
Reference: pp. 146 MPL 0.25

5. This method of measurement of blood flow considers a change in volume of the organ as equivalent to rate of blood flow:
A. Fick principle C. Ultrasonic flowmeter
B. Plethysmograph D. Electromagnetic flowmeter
Reference: pp. 147 MPL 0.50

6. The plateau observed in the cardiac action potential is due to:


A. Rapid depolarization due to increased sodium conductance
B. Closure of sodium channels and chloride influx
C. Slower but prolonged opening of voltage-gated potassium channels
D. Opening of voltage-gated potassium channels
Reference: pp. 97 MPL 0.75

7. The greater the ventricular walls are stretched during diastole, the greater is the force of contraction:
A. All-or-non Law C. Fick Principle
B. Frank-Starling Law D. Law of Laplace
Reference: pp. 103 MPL 1.00

8. The opening of the aortic valve occurs at the end of this phase of the cardiac cycle:
A. Maximum ejection C. Rapid inflow
B. Isovolumetric contraction D. Diastasis
Reference: pp. 103 MPL 0.25

9. The concept of preload refers to:


A. The degree to which the myocardium is stretched before it contracts
B. The resistance against which blood is expelled from the ventricles
C. The amount of blood pumped by each ventricle per minute
D. The effect of exercise on heart rate
Reference: pp. 103 MPL 0.75

10. What is the most important function of glucagons in the body when the blood sugar is low:
A. Activation of phosphorylase C. Gluconeogenesis in the liver
B. Glycogenolysis in liver tissue D. Release of fatty acid from liver
Reference: pp. 892 MPL 0.50

11. Vitamin D action includes the following, EXCEPT:


A. Regulates renal calcium ion transport
B. Inhibits proliferation and stimulates maturation of epidermal keratinocytes
C. Increase absorption of calcium in the stomach
D. Stimulate transcription of bone matrix protein osteocalcium
Reference: pp. 904 MPL 0.75

12. Which is NOT controlled by the anterior pituitary?


A. Testes C. Ovaries
B. Parathyroid D. Thyroid
Reference: pp. 846 MPL 1.00

13. Surgical removal of the pituitary gland results in all of the following condition, EXCEPT:
A. Salt loss and hypovolumic shock develops
B. Cold tolerated poorly
C. Sensitivity of stress
Physiology
2 / 50

D. Inhibition of growth
Reference: pp. 848 MPL 0.75

14. The rate of growth hormone secretion is affected by the following conditions, EXCEPT:
A. Starvation C. Glucocorticoid therapy
B. Hypoglycemia D. First 2 hours of deep sleep
Reference: pp. 852 MPL 0.50

15. G.G., 45-year old male office worker on his pre employment check up, showed marked enlargement of bone of the extremities and significant
coarsening of his facial feature. His jaw was likewise enlarged and protruding. The condition you would consider is:
A. Gigantism C. Acromegaly
B. Panhypopituitarism D. Dwarfism
Reference: pp. 854 MPL 1.00

16. High levels of glucocorticoids is associated with a decrease in this activity in the liver:
A. Glycogenolysis C. Glycogenesis
B. Gluconeogenesis D. Protein synthesis
Reference: pp. 876 MPL 0.25

17. The posterior pituitary gland stores the anti diuretic hormone. Which of the following is NOT TRUE of this hormone:
A. It is formed in the supra optic nuclei of the hypothalamus
B. It has an anti diuretic effect
C. It has a vasodilation effect on arterioles
D. It is necessary for osmotic regulation
Reference: pp. 855 MPL 0.75

18. Ella, 20 year old, female with an enlarged anterior neck mass causing dysphagia, no other signs and symptoms noted
TSH - normal
T3 - normal
T4 - normal

Your diagnosis is simple goiter. One of the following statements is true about her condition.
A. Her thyroid gland is synthesizing excess T3 & T4
B. Her symptoms arise from enlargement of thyroid causing mechanical compression
C. She will benefit from TSH injection
D. She will benefit from PTU
Reference: pp. 867 MPL 0.50

19. Sperm cells attain motility and capability to fertilize when they reach the:
A. Seminiferous tubules C. Epididymus
B. Vas deferens D. Seminal vesicles
Reference: pp. 918 MPL 0.50

20. Which of the following statements about spermatogenesis is CORRECT?


A. Sertoli cells are required for mitotic activity of germ cell
B. Spermatogenesis requires continuous release of gonadotropin releasing hormone
C. Production and release of spermatozoa is cyclical
D. Leydig cell secretion of testosterone requires follicle stimulating hormone
Reference: pp. 916 MPL 0.75

21. Pressure that the water molecules exert to escape through the surface:
A. Alveolar O2 pressure C. Coefficient of utilization
B. Pressure difference of gas D. Vapor pressure of water
Reference: pp. 453 MPL 1.00

22. The principal form by which carbon dioxide is transpsorted:


A. Carbaminohemoglobin C. Bicarbonate
B. Dissolved state D. Carbonic acid
Reference: pp. 470 MPL 1.00

23. This result from combination of oxygen and hemoglobin in the lungs causes the hemoglobin to be a stronger acid:
A. Haldane effect C. Oxygen, hemoglobin dissociation curve
B. Bohr effect D. Respiratory exchange ratio
Reference: pp. 471 MPL 0.50

24. The respiratory center in the medulla is strongly stimulated through the peripheral chemoreceptors when arterial:
A. pO2 falls below 60 mmHg C. H2 ion concentrates increases
B. pCO2 is increased D. pO2 is 100 mmHg
Reference: pp. 478 MPL 0.50

25. Strong signals to pneumotaxic center will:


A. Decrease the respiratory rate C. Increase gas exchange
B. Increase lung filling D. Shorten respiration
Reference: pp. 475 MPL 0.25

Physiology
3 / 50

26. One of the following statement is not true of carbon dioxide:


A. Its partial pressure in the athrospheric air is about 47 mmHg
B. Can diffuse 20x as rapidly in oxygen
C. Can be transported in dissolved state
D. Pulmonary capillary arterial and pressure is about 40 mmHg
Reference: pp. 459 MPL 0.50

27. When the blood become acidic the oxygen dissociation curve will likely show:
A. Shifting to the right
B. Shifting to the left
C. Lowered percentage of hemoglobin saturation
D. Increasing pO2 saturation
Reference: pp. 468 MPL 0.50

28. The part of the respiratory center the limits the duration of respiration and increases the respiratory rate:
A. Dorsal group of neurons C. Pneumotaxic center
B. Ventral D. Apneustic center
Reference: pp. 475 MPL 0.75

29. Respiratory distress syndrome in the newborn is least likely to be caused by:
A. Predominance of hemoglobin F in the newborn
B. Small diameter of the alveoli
C. Increased collapse pressure
D. Decreased surfactant
Reference: pp. 435 MPL 0.50

30. One of the following is not part of the respiratory membrane:


A. Alveolar epithelium
B. Epithelial basement membrane
C. Alveolar duct
D. Capillary basement membrane
Reference: pp. 459 MPL 0.75

31. Tidal volume plus the inspiratory reserve volume is the:


A. Vital capacity C. Functional residual capacity
B. Inspiratory capacity D. Total lung capacity
Reference: pp. 437 MPL 1.00

32. Which of the following statements regarding pressures in the pulmonary system is FALSE:
A. During systole pulmonary arterial pressure is equal to that of the right ventricular pressure
B. At the end of systole, the closure of the pulmonary valve produces an increase in the pulmonary arterial pressure
C. An increase of the left atrial pressure results to an increase of the pulmonary wege pressure
D. Left atrial pressure could be estimated by measuring pulmonary wedge pressure
Reference: pp. 447 MPL 0.50

33. The effect of decreased alveolar oxygen on local alveolar blood flow is:
A. Constriction
B. Decrease vascular resistance
C. No effect in blood flow
D. Damming of blood in the pulmonary circulation
Reference: pp. 445 MPL 0.50

34. If alveolar pressure is higher than the capillary blood pressure the pulmonary blood flow:
A. Is stopped C. Increases during diastole
B. Decreases during systole D. Is continous
Reference: pp. 446 MPL 0.50

35. Forces causing movement of fluid outward from the from the capillaries into the pulmonary interstitium include the following pressure
EXCEPT:
A. Capillary C. Negative interstitial fluid
B. Interstitial fluid colloid D. Plasma colloid osmotic
Reference: pp. 448 MPL 0.50

36. A 35 year old patient with scoliosis went to the emergency room because of difficulty of breathing. Pulmonary function test was done. What is
the expected result of the test?
A. Normal total lung capacity
B. Increased residual volume
C. Decreased maximum expiratory flow
D. Increased vital capacity
Reference: pp. 485 MPL 0.75

37. A 52 year old patient went to the OPD for consult. He has been smoking for the last 40 years consuming 1 pack a day. His complaint was
chronic cough with yellowish phlegm usually in the morning. This could be explained by:
A. Partial paralysis of the cilia of the respiratory epithelium
B. Destroyed normal protective mechanisms of the airways
Physiology
4 / 50

C. Inhibition of alveolar macrophages


D. All of the above
Reference: pp. 486 MPL 1.00

38. A chest x-ray was done to the above patient (chronic smoker) which revealed emphysema. What will be the physiologic effect on the patient?
A. Decreased airway resistance
B. Decreased physiologic dead space
C. Decreased diffusing capacity of the lungs
D. Pulmonary hypotension
Reference: pp. 486 MPL 0.50

39. O2 therapy is NOT effective in this type of hypoxia:


A. Atmospheric
B. hypoventilation
C. Hypoxial caused by impaired alveolar membrane diffusion
D. Hypoxia due to inadequate tissue use
Reference: pp. 490 MPL 0.50

40. The type of food that a person preferentially seeks is known as:
A. Hunger C. Mastication
B. Appetite D. Pica
Reference: pp. 728 MPL 1.00

41. The basic propulsive movement in the GIT is known as:


A. Mixing C. Peristalsis
B. Emulsification D. Spike contraction
Reference: pp. 733 MPL 1.00

42. The baglike bulges seen in the colon are known as:
A. Haustra C. Teniae coli
B. Ceca D. Tenia saginata
Reference: pp. 735 MPL 0.75

43. The distal opening of the stomach is the:


A. Antrum C. Cardia
B. Fundus D. Pylorus
Reference: pp. 730 MPL 1.00

44. After the food has been mixed in the stomach, the resulting mixture that passes out into the duodenum is known as:
A. Emollient C. Roughage
B. Miscelles D. Chyme
Reference: pp. 731 MPL 0.75

45. The mixing contractions known as segmentation are particularly seen in this segment of the GIT:
A. Esophagus C. Small intestines
B. Stomach D. Large intestines
Reference: pp. 734 MPL 1.00

46. Striated muscles are found in this segment of the esophagus:


A. First third C. Distal fluid
B. Middle third D. First half
Reference: pp. 730 MPL 1.00

47. Gastrointestinal contractions develop when the resting membrane potential of the GI smooth muscles become more positive than:
A. – 10 mmvolts C. – 30 mmvolts
B. – 20 mmvolts D. – 40 mmvolts
Reference: pp. 719 MPL 0.25

48. The volume of secreted bile is eventually increased after the addition of a watery solution of Na & HCO3 under the influence of this hormone:
A. Cholecystokinin C. Gastrin
B. Insulin D. Secretin
Reference: pp. 747 MPL 0.50

49. The submucosal plexus of the enteric nervous system controls:


A. Tone of the gut
B. Intensity of the rhythmical contractions
C. Contraction of submucosal muscle
D. Velocity of conduction of excitatory waves along the gut wall
Reference: pp. 721 MPL 0.50

50. A proteolytic enzyme which digests the fibrin fibers in a clot is:
A. Tissue plasminogen activator C. Plasmin
B. Plasminogen D. Profibrinolysins
Reference: pp. 425 MPL 0.50

Physiology
5 / 50

51. The initial event in the mechanism of hemostasis is:


A. Platelet plugging C. Clot formation
B. Vascular spasm D. Fibrous tissue formation into the clot
Reference: pp. 419 MPL 1.00

52. In the mechanism of clotting, this acts as an enzyme to convert fibrinogen into fibrin fibers:
A. Prothrombin C. Thrombin
B. Prothrombin activator D. ADP
Reference: pp. 421 MPL 1.00

53. The extrinsic pathway of clotting is initiated by:


A. Trauma to the blood C. Trauma to the tissues
B. Exposure of blood to collagen D. Loss of blood
Reference: pp. 423 MPL 1.00

54. This is an anticoagulant that removes thrombin in the blood:


A. Calcium C. Heparin
B. Citrate ions D. Oxalate ions
Reference: pp. 428 MPL 0.75

55. The initial event in the intrinsic pathway of clotting is:


A. Release of tissue factors
B. Activation of factor X
C. Activation of factor XI
D. Activation of factor XII and release of platelet phospholipids
Reference: pp. 423 MPL 0.25

56. Hemophilia B is caused by lack of this clothing factor:


A. Factor VIII C. Factor V
B. Factor I D. Factor IX
Reference: pp. 426 MPL 0.25

57. One of the constituents required for platelet aggregation is:


A. Collagen C. Thrombospondin
B. Fibronectin D. ADP
Reference: pp. 420 MPL 0.50

58. Vitamin K is necessary for the formation of this clotting factor


A. Factor V C. Factor VIII
B. Prothrombin D. Factor XI
Reference: pp. 421 MPL 1.00

59. Thromboembolic conditions in humans may be caused by:


A. Increased blood flow through the vessels
B. Roughened endothelial surface of a vessel
C. The presence of thrombomodulin in the vascular system
D. Presence of layer of glycocalyx absorbed to the inner surface of the endothelium
Reference: pp. 427 MPL 0.75

60. The hormone secreted mainly by the corpus luteum is:


A. Estrogen C. Testosterone
B. Progesterone D. Inhibin
Reference: pp. 936 MPL 0.75

61. Ovulation occurs:


A. Fourteen days before the onset of menstruation
B. Fourteen days after the onset of menstruation
C. Twelve days after the onset of menstruation
D. Twelve days before the onset of menstruation
Reference: pp. 931 MPL 0.50

62. The ovarian cycle is:


A. Atresia of the follicle
B. Increased secretion of progesterone
C. Accelerated growth of follicle
D. Involution of the corpus luteum
Reference: pp. 932 MPL 0.25

63. One of the following is NOT true with regards to the involution of the corpus luteum:
A. The final involution normally occurs exactly 12 days of corpus luteum life
B. Involution occurs 2 days before menstruation begins
C. Involution is caused by feedback effect mainly of progesterone on the anterior pituitary gland to maintain increased
secretion of FSH and LH
D. Involution is caused by secretion of inhibin which inhibits secretion of especially of FSH by the anterior pituitary gland
Physiology
6 / 50

Reference: pp. 932 MPL 0.25

64. The effect of estrogen on the uterus is:


A. It prevents expulsion of implanted ovum
B. It decreases the frequency and intensity of uterine contractions
C. It promotes secretory changes on the uterus during latter half of monthly female sexual cycle
D. It increases the motility of fallopian tubes and causes cervical mucus to be thinner and more alkaline
Reference: pp. 935 MPL 0.50

65. Fertilization of ovum takes place in what part of the fallopian tube:
A. Isthmus C. Ampulla
B. Timbria D. Cornu
Reference: pp. 944 MPL 0.75

66. implantation of fertilized ovum occurs:


A. on the 5th to 7th day after ovulation
B. on the 15th to 17th day after ovulation
C. on the 14th day after menstruation
D. on the 14th day after ovulation
Reference: pp. 945 MPL 0.75

67. The maximum secretion of human chorionic gonadotropin occurs on:


A. About the 10th – 12th week of pregnancy
B. About the 20th week of pregnancy
C. About the 6th to 7th week of pregnancy
D. About the 24th week of pregnancy
Reference: pp. 949 MPL 0.25

68. Ejection or “let down” process in milk secretion is the function of this hormone:
A. Prolactin C. Estrogen
B. Oxytocin D. Inhibin
Reference: pp. 956 MPL 1.00

69. One of the following is true with regards to physiologic changes in pregnancy:
A. There’s increase in the rate of water and electrolyte secretion in the urine due to increase glomerular filtration rate
B. Blood volume decreases due to hormonal effect of aldosterone and estrogen
C. The mother’s weight gain averages about 36 lbs
D. The metabolic rate of the mother does not change throughout pregnancy
Reference: pp. 950 MPL 0.75

70. Dyslexia means


A. Auditory aphasia C. Word deafness
B. Word blindness D. Global aphasia
Reference: pp. 667 MPL 1.00

71. Abilities combined to form working memory, EXCEPT:


A. Prognosticate C. Decreased aggressiveness
B. Plan for the future D. Solve complicated problems
Reference: pp. 672 MPL 0.25

72. Area for “Word formation”


A. Prefrontal association area C. Wernike’s area
B. Broca’s area D. Limbic area
Reference: pp. 636 MPL 0.50

73. Major reward centers have been found to be located on EXCEPT:


A. Medial forebrain bundle C. Ventromedial nuclei
B. Lateral hypothalamus D. Anterior pituitary gland
Reference: pp. 676 MPL 0.25

74. Strong stimulation of the punishment centers of the brain causes the following movements in animals EXCEPT:
A. Develop a denfense posture C. Lifting of tail
B. Extension of claus D. Narrowing of eyes
Reference: pp. 625 MPL 0.25

75. The acetylcholine is secreted by this neurons:


A. Raphe nuclei
B. Gigantocellular nervous of reticular excitatory area
C. Substancia Ingra
D. Locus ceruleus
Reference: pp. 519 MPL 0.25

76. One of the following can precipitate a grandmal attacks:


A. Rage C. Temperature = 37°C
B. PCO2 – 25 D. Benzodiazepines
Physiology
7 / 50

Reference: pp. 693 MPL 0.75

77. Brain wave the characterizes petit mal epilepsy:


A. Saw tooth C. Slow wave
B. Spike and dome pattern D. Asynchronous
Reference: pp. 694 MPL 0.75

78. Psychomotor seizure may present with the following features EXCEPT:
A. Short period of amnesia C. Moment of incoherent speech
B. Abnormal rage D. Diminished consciousness
Reference: pp. 694 MPL 0.50

79. Drugs that may increase the excitatory effects of norepinephrine and serotonin at nerve ending include:
A. Monoamine oxidase inhibitors C. Anticonvulsants
B. Anticholinergic D. Benzodiazepines
Reference: pp. 695 MPL 0.25

80. What will happen to a cell suspended in a hypotonic solution?


A. Swell C. Crenate
B. Shrink D. No change
Reference: pp. 271 MPL 0.50

81. The most important function of the kidney is:


A. Regulation of acid-base balance
B. Excretion of metabolic waste products
C. Regulation of arterial pressure
D. Secretion and metabolism of hormones
Reference: pp. 279 MPL 1.00

82. The sequence of renal blood flow from the renal artery towards the peritubular capillaries is as follows:
P. Interlobar artery T. Glomerulus
Q. Segmental artery U. Afferent artery
R. Interlobular artery V. Arcuate Artery
S. Efferent artery

A. P Q R S T U V C. V Q P S R T U
B. Q P R V U T S D. Q P V R U T S
Reference: pp. 282 MPL 0.25

83. The glomerular filtration rate is about_________ml/min


A. 105 C. 155
B. 125 D. 175
Reference: pp. 286 MPL 0.50

84. Which is NOT normally seen in the glomerular filtrate:


A. Sodium C. Protein
B. Water D. Potassium
Reference: pp. 284 MPL 0.75

85. Which of the following will increase GFR?


A. Decrease renal blood flow C. Kidney stones
B. Increase arterial pressure D. Renal disease
Reference: pp. 286 MPL 1.00

86. The solute concentration of the urine is high when:


A. ADH levels are high C. Cortisol levels are high
B. ADH levels are low D. Cortisol levels are low
Reference: pp. 315 MPL 0.75
87. Playing a set on tennis on a hot afternoon will:
A. Increase thirst and increase urine volume
B. Increase thirst and decrease urine volume
C. Decrease thirst and increase urine volume
D. Decrease thirst and decrease urine volume
Reference: pp. 325 MPL 1.00

88. This is a renal mechanism for concentrating the urine:


A. Aldosterone mechanism C. Renin angiotensin system
B. Countercurrent mechanism D. ADH-Thirst mechanism
Reference: pp. 317 MPL 1.00

89. The most powerful and the most complete regulation of acid-base balance is provided by the:
A. Chemical buffer system C. Kidneys
B. Respiratory system D. Endocrine system
Reference: pp. 281 MPL 1.00

Physiology
8 / 50

90. The kidney is able to regulate ECF hydrogen ions by the following mechanisms EXCEPT:
A. Secretion of hydrogen ions
B. Reabsorption of filtered bicarbonate
C. Production of new bicarbonate
D. Secretion of filtered bicarbonate
Reference: pp. 357 MPL 0.50

91. Which is NOT TRUE for T lymphocytes?


A. Differentiate and mature in the thymus
B. Establish residence in the paracortex area of the lymph node.
C. Provides the helper factor for B cell activation.
D. Differentiates into plasma cell.
Reference: pp. 403 MPL 0.50

92. Major feature of acquired immune response:


A. Nonspecific protection C. Immunologic memory
B. Protection provided by physical barrier D. Involves NK lymphocytes
Reference: pp. 402 MPL 0.75

93. TRUE statement regarding IgM:


A. Pentameric structure
B. Secreted on subsequent antigen exposure
C. Least numerous Immunoglobulin
D. Found in surface of mature virgin B cells
Reference: pp. 406 MPL 0.25

94. TRUE statement regarding IgG:


A. Dimer structure
B. Secreted on subsequent antigen exposure
C. Least numerous Immunoglobulin
D. Found in surface of mature virgin B cells
Reference: pp. 407 MPL 0.50

95. Which statement describes Secondary immune response:


A. Longer lag phase C. Rapid exponential phase
B. Lower peak state D. Involves IgM > IgG
Reference: pp. 406 MPL 1.00

96. The antibody present in the colostrums, saliva and tears is:
A. IgG C. IgM
B. IgA D. IgE
Reference: pp. 407 MPL 1.00

97. Helper T cells are recognized through this cell marker:


A. A. CD16+ C. CD4+CD-8
B. B. CD56+ D. CD4-CD8+
Reference: pp. 409 MPL 0.25

98. The Th profile that involves in atopic individual.


A. Th0 C. Th2
B. Th1 D. Th3
Reference: pp. 411 MPL 0.25

99. Lymph enters the lymph node through the:


A. Afferent lymphatic vessels C. Afferent arteriole
B. Efferent lymphatic vessels D. Efferent arteriole
Reference: pp. 396 MPL 0.50

100. This hypersensitivity reaction is associated with delayed type of reaction following exposure to eye cosmetics:
A. Type I C. Type III
B. Type II D. Type IV
Reference: pp. 411 MPL 0.50

1. The organ that removes the most abundant metabolic by-product of the body:
A. Liver C. Lungs
B. Kidney D. Gastrointestinal tract
ANS: C MPL: 0.5
REF: Guyton and Hall. Textbook of Medical Physiology. 9th ed., p. 5

2. Which of the following is an example of a negative feedback control mechanism?

Physiology
9 / 50

A. activation of clotting factors after rupture of blood vessel


B. contraction of uterine muscles during childbirth
C. generation of nerve signals after nerve fiber stimulation
D. stimulation of respiratory center during hypoxemia
ANS: D MPL: 0.25
REF: Guyton and Hall. Textbook of Medical Physiology. 9th ed., p. 8

CELL PHYSIOLOGY

3. The physical and chemical structure upon which protein molecules are assembled:
A. Endoplasmic reticulum C. Golgi apparatus
B. Mitochondria D. Ribosomes
ANS: D MPL: 0.5
REF: Guyton and Hall. Textbook of Medical Physiology. 9th ed., p. 31

4. Which of the following is a function of the golgi apparatus:


A. Protein synthesis C. Lipid synthesis
B. Formation of lysosomes D. Formation of ATP
ANS: B MPL: 0.5
REF: Guyton and Hall. Textbook of Medical Physiology. 9th ed., p. 15

5. The following are functions of membrane proteins, EXCEPT:


A. Structural channels for diffusion of water
B. Catalyze chemical reactions
C. Cell to cell attachment
D. Carriers for active transport of substances
ANS: C MPL: 0.5
REF: Guyton and Hall. Textbook of Medical Physiology. 9th ed., p. 14

TRANSPORT PROCESSES

6. Which of the following transport processes will NOT require the direct input of energy in the form of ATP:
A. movement of sodium out of the cell
B. movement of glucose into muscle cells
C. movement of calcium into the sarcoplasmic reticulum
D. movement of hydrogen ion into the lumen of the distal nephron
ANS: B MPL: 0.33
REF: Guyton and Hall. Textbook of Medical Physiology. 9th ed., p. 48

7. Inhibition of the sodium-potassium pump can be expected to increase which of the following:
A. negativity of the membrane potential C. membrane excitability
B. cell volume D. ECF sodium concentration
ANS: B MPL: 0.33
REF: Guyton and Hall. Textbook of Medical Physiology. 9th ed., p. 52

8. Diffusion of a positively charged substance out of the cell is increased when there is an increase in:
A. positively charged ions outside the cell
B. concentration of the substance inside the cell
C. pressure outside the cell
D. negativity inside the cell
ANS: B MPL: 0.5
REF: Guyton and Hall. Textbook of Medical Physiology. 9th ed., pp. 48-49

GENETIC BASIS FOR CELLULAR FUNCTION

9. Cell differentiation is the result of:


A. loss of genetic material
B. mutation
C. failure of DNA proof-reading
D. selective repression of operons
ANS: D MPL: 0.33
REF: Guyton and Hall. Textbook of Medical Physiology. 9th ed., p. 37-38

10. TRUE of the process of transcription:


A. Both strands of DNA are replicated
B. Results in the synthesis of mRNA
C. DNA polymerase is the principal enzyme
D. Amino acid sequence is determined by the anticodons of tRNA
ANS: B MPL: 0.5
REF: Guyton and Hall. Textbook of Medical Physiology. 9th ed., pp. 29-30

MEMBRANE ELECTROPHYSIOLOGY

11. A drug completely blocks voltage-gated Na+ channels in nerves. Which of the following effects on the action potential would it be expected to
produce:
Physiology
10 / 50

A. Blocks the occurrence of action potentials


B. Increase the rate of rise of the upstroke of the action potential
C. Shorten the absolute refractory period
D. Abolish the hyperpolarizing after potential
ANS: A MPL: 0.33
REF: Guyton and Hall. Textbook of Medical Physiology. 9th ed., pp. 61-65, 70

12. The repolarization phase of the action potential is due to:


A. diffusion of chloride into the cell
B. diffusion of potassium out of the cell
C. diffusion of sodium into the cell
D. activation of the sodium-potassium pump
ANS: B MPL: 0.5
REF: Guyton and Hall. Textbook of Medical Physiology. 9th ed., p. 62

13. The hyperpolarization of a membrane after an action potential is due to:


A. rapid entry of Na+ into the cell
B. rapid entry of Ca2+
C. slow closure of K+ channels
D. rapid closure of Na+ channels
ANS: C MPL: 0.5
REF: Guyton and Hall. Textbook of Medical Physiology. 9th ed., p. 64

14. Increasing the magnitude of the stimulus above threshold will produce which of the following:
A. Increased amplitude of the action potential
B. Increased velocity of propagation of the action potential
C. Increased duration of the action potential
D. No appreciable change in the action potential
ANS: D MPL: 0.5
REF: Best and Taylor’s Physiological Basis of Medical Practice, 12th ed., p. 45

15. The resting membrane potential is determined largely by the:


A. potassium efflux through the voltage-gated channels
B. sodium influx through the voltage-gated channels
C. potassium efflux through the Na+-K+ leak channels
D. sodium influx through the Na+-K+ leak channels
ANS: C MPL: 0.5
REF: Guyton and Hall. Textbook of Medical Physiology. 9th ed., pp. 60-61

SYNAPTIC TRANSMISSION

16. The most common mechanism for synaptic excitation:


A. Opening of chloride channels
B. Increase number of excitatory receptors
C. Opening of sodium channels
D. Increased negativity of the cell interior
ANS: C MPL: 0.5
REF: Guyton and Hall. Textbook of Medical Physiology. 9th ed., p. 572

17. Release of neurotransmitters from the nerve terminal by exocytosis would be blocked most effectively by preventing the:
A. propagation of the action potential into the nerve terminal membrane
B. flow of Na+ into the nerve terminal membrane
C. flow of K+ out of the nerve terminal membrane
D. flow of calcium into the nerve terminal membrane
ANS: D MPL: 0.5
REF: Guyton and Hall. Textbook of Medical Physiology. 9th ed., p. 570

18. Which of the following will determine whether the release of a neurotransmitter at the synaptic junction will result in excitation or inhibition of a
postsynaptic neuron:
A. The chemical structure of the neurotransmitter
B. The properties of the postsynaptic receptor
C. The rate of reuptake of the neurotransmitter by the presynaptic cell
D. The amount of calcium released from the presynaptic axons
ANS: B MPL: 0.5
REF: Guyton and Hall. Textbook of Medical Physiology. 9th ed., pp. 570

19. Which of the following neurotransmitters is found in areas of the brain responsible for memory and long term behavior?
A. Glycine C. Aspartate
B. Nitric Oxide D. Glutamate
ANS: B MPL: 0.25
REF: Guyton and Hall. Textbook of Medical Physiology. 9th ed., p. 573

MUSCLE PHYSIOLOGY

20. Which of the following is TRUE of smooth muscle contraction compared to that of skeletal muscle:
Physiology
11 / 50

A. Onset of contraction is faster


B. Frequency of crossbridge cycling is faster
C. Energy required to sustain a contraction is greater
D. Force of contraction is greater
ANS: D MPL: 0.33
REF: Guyton and Hall. Textbook of Medical Physiology. 9th ed., p. 97

21. Which of the following is a characteristic of fast fatigable skeletal muscle fibers also known as white fibers:
A. High myobin content C. High mitochondrial content
B. High glycolytic activity D. High capillary network density
ANS: B MPL: 0.5
REF: Best and Taylor’s Physiological Basis of Medical Practice, 12th ed., p. 100, Table 1.9

22. Which of the following temporal sequences is CORRECT for excitation-contraction coupling in skeletal muscle:
A. Release of calcium from the SR; action potential in the muscle membrane; crossbridge formation
B. Action potential in the muscle membrane; depolarization of the T – tubule; release of calcium from the SR
C. Action potential in the muscle membrane; hydrolysis of ATP; binding of calcium to troponin C
D. Release of calcium from the SR; depolarization of the T – tubule; binding of calcium to troponin C
ANS: B MPL: 0.5
REF: Best and Taylor’s Physiological Basis of Medical Practice, 12th ed., p. 86

23. A person with myasthenia gravis notes increased muscle strength when he is treated with an acetylcholinesterase inhibitor. The basis for his
improvements is increased:
A. amounts of acetylcholine released from the motor nerves
B. levels of acetylcholine at the motor end plates
C. number of acetylcholine receptors in the motor end plate
D. amount of norepinephrine released from motor nerves
ANS: B MPL: 0.33
REF: Guyton and Hall. Textbook of Medical Physiology. 9th ed., p. 91

24. An isotonic contraction differs from an isometric contraction in that in an isotonic contraction:
A. the muscle is less efficient
B. the muscle uses more ATP
C. the heat of activation (initial heat) is greater
D. the recovery heat is reduced
ANS: B MPL: 0.33
REF: Best and Taylor’s Physiological Basis of Medical Practice, 12th ed., p. 87

25. Which of the following proteins is important for skeletal muscle contraction but not for smooth muscle contraction:
A. Actin C. Troponin
B. Myosin D. Ca2+ - ATPase
ANS: C MPL: 0.5
REF: Guyton and Hall. Textbook of Medical Physiology. 9th ed., p. 96

BONE PHYSIOLOGY

26. Which component of bone tissue is responsible for its compressional strength?
A. Chondroitin sulfate . Hydroxyapatite
B. Collagen D. Hyaluronic acid
ANS: C MPL: 0.5
REF: Guyton and Hall. Textbook of Medical Physiology. 9th ed., p. 989

27. TRUE of bone resorption:


A. A function of the osteoblasts
B. Associated with increased alkaline phosphatase concentration in plasma
C. Bone salts are dissolved by acids secreted by phagocytic cells
D. Involves a piezoelectric effect mechanism
ANS: C MPL: 0.5
REF: Guyton and Hall. Textbook of Medical Physiology. 9th ed., pp. 990-991

BODY FLUIDS PHYSIOLOGY

28. Which of the following can be expected to occur if an individual drank 6 liters of purified water rapidly assuming no further loss of water from his body
occurs:
A. Osmolality of total body water will increase
B. ECF volume will decrease by 2 liters
C. Osmotically active solute concentration in the ICF will increase
D. ICF volume will increased by 4 liters
ANS: D MPL: 0.5
REF: Best and Taylor’s Physiological Basis of Medical Practice, 12th ed., pp 416-417

29. TRUE of the interstitial fluid compartment:


A. It contains more anions than cations
B. It contains more cations than plasma
C. Its total concentration of ions is less than that of the ICF
Physiology
12 / 50

D. Its osmolality is greater than that of plasma


ANS: C MPL: 0.33
REF: Best and Taylor’s Physiological Basis of Medical Practice, 12th ed., pp 410-411

30. A cell placed in an unknown solution was noted to shrink in size. Which of the following will be TRUE of the solution:
A. Osmolality of the solution is higher than that of the ECF
B. Solute concentration of the solution is lower than that of the ICF
C. Solutes in the solution are freely permeable through the cell membrane
D. Water concentration is greater than solute concentration
ANS: A MPL: 0.5
REF: Guyton and Hall. Textbook of Medical Physiology. 9th ed., p. 304

PHYSIOLOGY OF THE HEART

31. The primary controller of cardiac output:


A. Stroke volume C. Venous return
B. Heart rate D. Systemic arterial pressure
ANS: C MPL: 0.5
REF: Guyton and Hall. Textbook of Medical Physiology. 9th ed., p. 240

32. The Frank-Starling’s Law of the Heart is manifested by which of the following conditions:
A. Increase in heart rate when venous return is increased
B. Increase in stroke volume when ventricular end-diastolic volume is increased
C. Increase in ventricular end-diastolic pressure when aortic pressure is increased
D. Increase in cardiac output when heart rate is increased
ANS: B MPL: 0.5
REF: Guyton and Hall. Textbook of Medical Physiology. 9th ed., p. 240

33. The most important determinant of coronary blood flow:


A. Coronary perfusion pressure
B. Systolic compression of coronary vessels
C. Myocardial oxygen consumption
D. Level of sympathetic tone
ANS: C MPL: 0.5
REF: Best and Taylor’s Physiological Basis of Medical Practice, 12th ed., p. 265

34. The sinoatrial node is the normal pacemaker of the heart because:
A. it has both sympathetic and parasympathetic nerve supply
B. it is the only one capable of spontaneous diastolic depolarization
C. it has the fastest velocity of impulse conduction
D. it has the highest frequency of impulse generation
ANS: D MPL: 0.5
REF: Guyton and Hall. Textbook of Medical Physiology. 9th ed., p. 125

35. Spontaneous diastolic depolarization of cardiac pacemaker cells is due to the gradual decrease in the membrane permeability of which of the
following ions:
A. Sodium C. Calcium
B. Potassium D. Chloride
ANS: B MPL: 0.5
REF: Best and Taylor’s Physiological Basis of Medical Practice, 12th ed., p. 161

36. TRUE of the cardiac cycle in the right and left side of the heart:
A. Isovolumetric contraction is shorter in the right ventricle than in the left ventricle
B. Right ventricle contracts earlier than the left ventricle
C. Period of ejection is longer in the left ventricle than in the right venticle
D. Atrial systole occurs earlier in the left atrium than in the right atrium
ANS: A MPL: 0.5
REF: Best and Taylor’s Physiological Basis of Medical Practice, 12th ed., p. 241

37. Acetylcholine will increase the membrane permeability of the SA node to:
A. sodium C. potassium
B. chloride D. calcium
ANS: C MPL: 0.5
REF: Guyton and Hall. Textbook of Medical Physiology. 9th ed., p. 126

PHYSIOLOGY OF THE CIRCULATORY SYSTEM

38. Which of the following is a long-term arterial pressure control mechanism:


A. Capillary fluid shift mechanism C. Baroreceptor reflex
B. Pressure diuresis D. Renin-Angiotensin system
ANS: B MPL: 0.5
REF: Guyton and Hall. Textbook of Medical Physiology. 9th ed., p. 221

39. Which of the following will be true when the resistance in a circulatory bed is arranged parallel to each other:
A. Total resistance will be higher than any of the individual resistance in the circuit
Physiology
13 / 50

B. A decrease in one of the resistance will decrease total conductance


C. The addition of another resistance to the circuit will decrease total resistance
D. Total resistance will equal the sum of all the individual resistance in the circuit
ANS: C MPL: 0.33
REF: Best and Taylor’s Physiological Basis of Medical Practice, 12th ed., pp. 141-142

40. Peripheral chemoreceptors will respond to which of the following stimuli:


A. Decreased carbon dioxide concentration
B. Decreased arterial pressure
C. Decreased oxygen concentration
D. Decreased hydrogen ion concentration
ANS: C MPL: 0.33
REF: Guyton and Hall. Textbook of Medical Physiology. 9th ed., p. 216

41. The highest proportion of stored oxygen in the body is derived from:
A. oxygen dissolved in body fluids
B. oxygen combined with hemoglobin in the blood
C. oxygen in the air inside the lungs
D. oxygen combined with myoglobin in muscles
ANS: B MPL: 0.5
REF: Guyton and Hall. Textbook of Medical Physiology. 9th ed., p. 1062

42. The most important factor that regulates the opening and closing of the metarterioles and precapillary sphincters is the:
A. degree of sympathetic stimulation C. tissue oxygen concentration
B. level of arterial pressure D. velocity of blood flow
ANS: C MPL: 0.5
REF: Guyton and Hall. Textbook of Medical Physiology. 9th ed., pp. 184-185

43. Which of the following is decreased as a compensatory mechanisms for heart failure:
A. atrial natriuretic factor C. aldosterone
B. glomerular filtration rate D. catecholamines
ANS: B MPL: 0.5
REF: Guyton and Hall. Textbook of Medical Physiology. 9th ed., p. 266

44. A drug capable of increasing arterial pressure without causing any change in the cardiac output is most likely to exert this effect through which of
the following mechanisms:
A. Increased venous return C. Increased heart rate
B. Increased peripheral resistance D. Increased mean systemic filling pressure
ANS: B MPL: 0.5
REF: Best and Taylor’s Physiological Basis of Medical Practice, 12th ed., p. 316

45. Which of the following Starling’s forces will favor the reabsorption of fluid back into the capillaries under normal conditions:
A. Capillary hydrostatic pressure C. Interstitial fluid hydrostatic pressure
B.Plasma colloid osmotic pressure D. Interstitial fluid colloid osmotic pressure
ANS: B MPL: 0.5
REF: Guyton and Hall. Textbook of Medical Physiology. 9th ed., p. 187-189, 191-192

RENAL PHYSIOLOGY

46. Which of the following factors will increase tubular reabsorption:


A. Increased glomerular filtration rate
B. Decreased filtration coefficient of peritubular capillaries
C. Increased peritubular capillary hydrostatic pressure
D. Decreased afferent arteriolar resistance
ANS: A MPL: 0.5
REF: Guyton and Hall. Textbook of Medical Physiology. 9th ed., pp. 341-343

47. The high osmolality of the renal medullary interstitium is mostly due to:
A. Active transport of ions from the collecting ducts into the medullary interstitium
B. Passive diffusion of urea from the collecting ducts into the medullary interstitium
C. Impermeability of the medullary tubules to water
D. Active reabsorption of solutes from the thick ascending limb of Henle’s Loop
ANS: D MPL: 0.33
REF: Guyton and Hall. Textbook of Medical Physiology. 9th ed., p. 352

48. Which of the following sets of arterial blood gas values would be consistent with an acid-base disorder found in a patient with profuse diarrhea:
pH pCO2 HCO3
A. 7.48 33 24
B. 7.35 35 20
C. 7.49 48 35
D. 7.20 78 30
ANS: B MPL: 0.5
REF: Guyton and Hall. Textbook of Medical Physiology. 9th ed., p. 400

49. The major site for autoregulatory resistance changes in response to changes in arterial pressure:
Physiology
14 / 50

A. Renal artery C. Glomerular capillary


B. Afferent arteriole D. Efferent arteriole
ANS: B MPL: 0.5
REF: Vander AJ., Renal Physiology, 5th ed., p. 34

50. Which of the following will increase hydrogen ion secretion:


A. Decreased plasma potassium concentration
B. Decreased sodium reabsorption
C. Decreased aldosterone secretion
D. Decreased arterial pCO2
ANS: A MPL: 0.5
REF: Best and Taylor’s Physiological Basis of Medical Practice, 12th ed., pp.496-497

51. Which of the following conditions will increase GFR:


A. Increased afferent arteriolar resistance
B. Increased Bowman’s capsule hydrostatic pressure
C. Increased glomerular capillary colloid osmotic pressure
D. Increased efferent arteriolar resistance
ANS: D MPL: 0.5
REF: Guyton and Hall. Textbook of Medical Physiology. 9th ed., p. 325, Table 26-2

52. A drug that blocks the angiotensin II receptor can be expected to increase which of the following:
A. sodium reabsorption
B. aldosterone formation
C. plasma potassium concentration
D. blood pressure
ANS: C MPL: 0.5
REF: Guyton and Hall. Textbook of Medical Physiology. 9th ed., pp. 371-372

53. Diabetes insipidus due to an absolute deficiency of ADH would be expected to manifest which of the following:
A.Decrease in urine volume
B.Negative free water clearance
C.Low urine osmolality
D.Increased urinary sodium excretion
ANS: C MPL: 0.5
REF: Guyton and Hall. Textbook of Medical Physiology. 9th ed., p. 349-350

54. The bicarbonate buffer system is the most important buffer system in the body because:
A.bicarbonate is found in high concentration in plasma
B.its pKa is very close to plasma pH
C.it is the most effective among all chemical buffer systems in the body
D.it can be regulated by the lungs and the kidney
ANS: D MPL: 0.5
REF: Guyton and Hall. Textbook of Medical Physiology. 9th ed., p. 389

55. A patient with abnormally low levels of aldosterone would be expected to manifest which of the following findings:
A. Hypertension C. High plasma potassium
B. Metabolic alkalosis D. High plasma sodium
ANS: C MPL: 0.33
REF: Guyton and Hall. Textbook of Medical Physiology. 9th ed., p. 372

56. The amount of potassium excreted by the kidney will decrease if:
A.distal tubular flow increases
B.dietary intake of potassium increases
C.sodium reabsorption by the distal nephron decreases
D.the excretion of organic anions increases
ANS: C MPL: 0.5
REF: Best and Taylor’s Physiological Basis of Medical Practice, 12th ed., p. 507, Table 4.25

ENDOCRINE PHYSIOLOGY

57. A deficiency in insulin can be expected to increase:


A. protein synthesis C. fatty acid synthesis in the liver
B. glycolysis D. glycogenolysis
ANS: D MPL: 0.5
REF: Guyton and Hall. Textbook of Medical Physiology. 9th ed., pp. 973 - 976

58. Cardiovascular effects of thyroid hormone will include an increase in all of the following, EXCEPT:
A.heart rate C. cardiac output
B.mean arterial pressure D. pulse pressure
ANS: B MPL: 0.5
REF: Guyton and Hall. Textbook of Medical Physiology. 9th ed., p. 950

59. Growth hormone will DECREASE which of the following:

Physiology
15 / 50

A. Uptake of glucose into the cell


B. Mobilization of fat from adipose tissue
C. Amino acid transport into the cell
D. Transcription of DNA to RNA
ANS: A MPL: 0.5
REF: Guyton and Hall. Textbook of Medical Physiology. 9th ed., p. 937

60. Which of the following is NOT a function of cortisol:


A.increases peripheral glucose utilization
B.reduces protein stores in cells except in the liver
C.stimulates gluconeogenesis by the liver
D.promotes mobilization of fatty acids from adipose tissue
ANS: A MPL: 0.33
REF: Guyton and Hall. Textbook of Medical Physiology. 9th ed., p. 962 - 963

61. Somatostatin secretion is inhibited by:


A.glucose C. epinephrine
B.amino acids D. acetylcholine
ANS: C MPL: 0.5
REF: Best and Taylor’s Physiological Basis of Medical Practice, 12th ed., p. 765

62. The principal target organs of PTH are:


Bone & Intestines C. Kidney & Intestines
Bone & Kidney D. Kidney & Immune system
ANS: B MPL: 1.0
REF: Guyton and Hall. Textbook of Medical Physiology. 9th ed., pp. 992 - 993

REPRODUCTIVE PHYSIOLOGY

63. Which of the following is a physiologic action of progesterone:


A. Promote proliferative changes in the endometrium
B. Increased frequency and intensity of uterine contraction
C. Stimulate the development of lobules and alveoli in the breast
D. Increased osteoblastic activity
ANS: C MPL: 1.0
REF: Guyton and Hall. Textbook of Medical Physiology. 9th ed., p. 1024 - 1025

64. The hormone that stimulates testosterone secretion in the male:


A. growth hormone C. estrogen
B. FSH D. LH
ANS: D MPL: 0.5
REF: Guyton and Hall. Textbook of Medical Physiology. 9th ed., p. 1013

65. Which of the following phases of the female sexual response is mediated by the parasympathetic nervous system:
A.stimulation C. lubrication
B.orgasm D. resolution
ANS: C MPL: 0.33
REF: Guyton and Hall. Textbook of Medical Physiology. 9th ed., p. 1030

BLOOD PHYSIOLOGY

66. NO AGGLUTINATION will occur if you mix RED BLOOD CELLS of BLOOD TYPE:
A. A with serum of type O
B. B with serum of type AB
C. A with serum of type B
D. B with serum of type O
ANS: B MPL: 0.25
REF: Guyton and Hall. Textbook of Medical Physiology, 9th ed., p. 459

67. The cells responsible for acquired immune response are:


A. basophils C. lymphocytes
B. eosinophils D. neutrophils
ANS: C MPL: 1
REF: Guyton and Hall. Textbook of Medical Physiology, 9th ed., p. 446

68. The primary function of antibodies is to:


A. act as opsonins
B. activate complement system
C. bind antigen to lymphocytes
D. cause antigen to clump
ANS: C MPL: 0.25
REF: Guyton and Hall. Textbook of Medical Physiology, 9th ed., p. 448

69. Extrinsic pathway of blood coagulation is initiated by:


A. activation of antigen-antibody reaction
Physiology
16 / 50

B. contact of blood with vessel wall


C. injury to vascular wall
D. trauma to blood
ANS: C MPL: 0.5
REF: Guyton and Hall. Textbook of Medical Physiology, 9th ed., p. 466

70. The classical pathway of complement system is activated by:


A. antigen-antibody reaction
B. migration of leukocytes
C. phagocytosis of microorganisms
D. polysaccharide in cell membrane
ANS: A MPL: 0.5
REF: Guyton and Hall. Textbook of Medical Physiology, 9th ed., p. 450

GI PHYSIOLOGY

71. Gastric emptying is stimulated by:


A. duodenal distention C. gastrin secretion
B. duodenal irritation D. secretin secretion
ANS: C MPL: 1
REF: Guyton and Hall. Textbook of Medical Physiology, 9th ed., p. 807

72. Choleystokinin stimulates:


A. gallbladder contraction C. pancreatic secretion
B. gastric emptying D. pepsinogen secretion
ANS: A MPL: 1
REF: Guyton and Hall. Textbook of Medical Physiology, 9th ed., p. 798

73. One of the following processes is INCREASED by sympathetic stimulation:


A. gastric secretion C. pancreatic secretion
B. intestinal secretion D. salivary secretion
ANS: D MPL: 1
REF: Guyton and Hall. Textbook of Medical Physiology, 9th ed., p. 819

74. Which of the following processes is important for digestion?


A. gastric secretion C. pancreatic secretion
B. intestinal secretion D. salivary secretion
ANS: C MPL: 0.5
REF: Guyton and Hall. Textbook of Medical Physiology, 9th ed., p. 835

75. Removal of the ileum will interfere with absorption of:


A. carbohydrates B. iron C. vitamin B12 D. water
ANS: C MPL: 1
REF: Guyton and Hall. Textbook of Medical Physiology, 9th ed., p. 820

76. All of the following processes require pancreatic secretion EXCEPT:


A. carbohydrate digestion C. electrolyte absorption
B. fat absorption D. protein digestion
ANS: C MPL: 1
REF: Guyton and Hall. Textbook of Medical Physiology, 9th ed., p. 824

77. During swallowing, entry of food into the trachea is prevented by:
A. closure of the glottis C. elevation of soft palate
B. elevation of the larynx D. relaxation of upper esophageal sphincter
ANS: A MPL: 1
REF: Guyton and Hall. Textbook of Medical Physiology, 9th ed., p. 804

78. The most important pancreatic enzyme needed for protein digestion is:
A. aminopeptidase C. chymotrypsin
B. carboxypeptidase D. trypsin
ANS: D MPL: 1
REF: Guyton and Hall. Textbook of Medical Physiology, 9th ed., p. 835

79. Defecation reflex is initiated by:


A. contraction of internal anal sphincter C. inhibition of parasympathetic nerves
B. distention of rectal wall D. stimulation of sympathetic nerves
ANS: B MPL: 0.5
REF: Guyton and Hall. Textbook of Medical Physiology, 9th ed., p. 812

80. The hormone that stimulates pancreatic and hepatic HCO3-secretion is:
A. cholecystokinin B. gastrin C. GIP D. Secretin
ANS: D MPL: 1
REF: Guyton and Hall. Textbook of Medical Physiology, 9th ed., p. 825

Physiology
17 / 50

PULMONARY PHYSIOLOGY

81. During quiet breathing, work is performed primarily to:


A. expand chest wall and lungs. C. recoil chest wall and lungs.
B. overcome airway resistance. D. resist viscosity of chest wall and lungs.
ANS: A MPL: 0.5
REF: Guyton and Hall. Textbook of Medical Physiology, 9th ed., p. 481

82. Work of breathing is increased by:


A. decreasing length of airways C. increasing diameter of airways
B. decreasing tissue elastic force D. increasing surface tension elastic force
ANS: D MPL: 0.25
REF: Guyton and Hall. Textbook of Medical Physiology, 9th ed., p. 481

83. The lung volume that provides air in the alveoli to aerate blood between breaths is known:
A. tidal volume. C. expiratory reserve volume.
B. inspiratory reserve volume. D. residual volume.
ANS: D MPL: 1
REF: Guyton and Hall. Textbook of Medical Physiology, 9th ed., p. 482

84. Pulmonary edema decreases the rate of diffusion of gases by INCREASING:


A. pressure of alveolar gases C. surface area of respiratory membrane
B. solubility of gases D. thickness of respiratory membrane
ANS: D MPL: 0.5
REF: Guyton and Hall. Textbook of Medical Physiology, 9th ed., p. 508

85. Wasted ventilation occurs when:


A. perfusion and ventilation are adequate.
B. perfusion is adequate, ventilation is absent.
C. perfusion is absent, ventilation is adequate.
D. perfusion and ventilation are absent.
ANS: B MPL: 0.5
REF: Guyton and Hall. Textbook of Medical Physiology, 9th ed., p. 511

86. Shunting of blood occurs when:


A. perfusion and ventilation are adequate.
B. perfusion is adequate, ventilation is absent.
C. perfusion is absent, ventilation is adequate.
D. perfusion and ventilation are absent.
ANS: C MPL: 0.5
REF: Guyton and Hall. Textbook of Medical Physiology, 9th ed., p. 510

87. The affinity of oxygen with hemoglobin decreases during:


A. hypothermia. C. polycythemia.
B. metabolic acidosis. D. respiratory alkalosis.
ANS: B MPL: 0.3
REF: Guyton and Hall. Textbook of Medical Physiology, 9th ed., p. 518

88. Increasing the tidal volume INCREASES:


A. functional residual capacity C. inspiratory capacity
B. expiratory reserve volume D. residual volume
ANS: C MPL: 0.5
REF: Guyton and Hall. Textbook of Medical Physiology, 9th ed., p. 482

89. One of the following is a physiologic response to high altitude:


A. decreased minute ventilation
B. decreased pulmonary vascular resistance
C. increased tubular reabsorption of HCO3-
D. increased diffusing capacity
ANS: D MPL: 0.5
REF: Guyton and Hall. Textbook of Medical Physiology, 9th ed., p. 551

90. Mechanism of decompression sickness:


A. compression of gas in lungs during immersion
B. expansion of gas in lungs during breath-holding
C. dissolution of gas in solution during descent to deep sea
D. formation of gas bubbles during rapid ascent to sea level
ANS: D MPL: 0.25
REF: Guyton and Hall. Textbook of Medical Physiology, 9th ed., p. 560

NEUROPHYSIOLOGY

91. The prolonged inhibitory post-synaptic potential (slow IPSP) seen in sympathetic ganglia is mediated by which of the following types of receptors
A. cholinergic
B. dopaminergic
Physiology
18 / 50

C. Nicotinic
D. Muscarinic
ANS: B MPL: 0.25
REF: Ganong. Review of Med Physiology, Chap. 13, p. 177

92. Bronchial muscle relaxation involves which of the following receptor type
A. cholinergic
B. alpha nradrinergic
C. beta-1 noradrinergic
D. beta-2 noradrinergic
ANS: D MPL: 1.0
REF: Ganong. Review of Med Physiology, Chap. 13, p. 176

93. The temperature or the scrotum is regulated carefully at what temperature


A. 30o C
B. 32o C
C. 34o C
D. 36o C
ANS: B MPL: 0.25
REF: Ganong. Review of Med Physiology, Chap. 14, p. 195

94. Which of the following tracts is expected to terminate in the flocculonodular lobe
A. dorsal spinocereballar
B. ventral spinocerebellar
C. pontocerebellar
D. vestibulocerebellar
ANS: D MPL: 0.5
REF: Ganong. Review of Med Physiology, Chap. 12, p. 169

95. Adult human brainwaves noted on an EEG at rest and having a frequency of 8-12/s are known as which of the following
A. alpha rhythm
B. beta rhythm
C. delta rhythm
D. theta rhythm
ANS: A MPL: 0.5
REF: Ganong. Review of Med Physiology, Chap. 11, p. 148

96. In the Young-Helmholtz Theory, color vision determination is subserved by how many different types of main cone color systems
A. 2
B. 3
C. 4
D. 6
ANS: B MPL: 0.25
REF: Ganong. Review of Med Physiology, Chap. 8, p. 125

97. Which of the following statements regarding the amygdala is TRUE:


A. It is a complex of nuclei located beneath the cortex of the parietal lobe
B. It receives neuronal signals almost solely from the cerebellum
C. Stimulation may cause either an increase or decrease of arterial pressure
D. Acts as an intermediate pathway for sensory information between the spinal cord and the Thalamus
ANS: C MPL: 0.25
REF: Guyton. Textbook of Medical Physiology, 8th ed., Chap 58, p. 656

98. The auditory pathways will involve which of the following structures
A. medial lemniscus and superior colliculus
B. lateral leminiscus and superior colliculus
C. medial lemniscus and inferior colliculus
D. lateral leminiscus and inferior colliculus
ANS: C MPL: 0.25 1/4
REF: Guyton. Textbook of Medical Physiology, 8th ed., Chap 52, p. 576

99. The area of the hypothalamus mainly involved with vasopressin release
A. supraoptic nucleus
B. posterior pre-optic nucleus
C. medial preoptic nucleus
D. perifornical nucleus
ANS: A MPL: 0.25
REF: Guyton. Textbook of Medical Physiology, 8th ed., Chap 58, p. 653

100. The approximate rate of CSF formations per day is


A. 0.5 liter
B. 1.0 liter
C. 2.0 liters
D. 3.0 liters
ANS: A MPL: 0.25
Physiology
19 / 50

REF: Guyton. Textbook of Medical Physiology, 8th ed., Chap. 61, p. 681

CELL
C 1. Substances with high diffusion coefficient:
A. include large polar molecules
B. include sodium and potassium ions
C. are more soluble in lipid than in water
D. usually cross the cell membrane by active transport
* Textbook of Medical Physiology 10th ed, by Guyton and Hall

B 2. Membrane proteins :
A. act as transporters of hydrophobic molecules across the membrane
B. serve as the transducing sites to conduct messages across the cell membrane
C. are located only at the external surface of the cell membrane
D. transport only the small polar substances across the cell membrane
* Textbook of Medical Physiology 10th ed, by Guyton and Hall

B 3. Glycolcalyx is found on the outer cell membrane surface. This:


A. is usually positively charged
B. binds with extracellular calcium to stabilize membrane structures
C. is the resulting lipid layer on the external cellular surface
D. results from the binding of carbohydrate with membrane proteins
* Textbook of Medical Physiology 10th ed, by Guyton and Hall

C 4. Which of the following substances is/are synthesized on the surface of the endoplasmic reticulum by the interaction of messenger RNA with
ribosomes?
A. lipids B. carbohydrates C. proteins D. all of these
* Textbook of Medical Physiology 10th ed, by Guyton and Hall

B 5. What organelle processes the endoplasmic reticulum products for final translocation to the plasma
membrane?
A. mitochondria B. Golgi apparatus C. ribosomes D. microtubule
* Textbook of Medical Physiology 10th ed, by Guyton and Hall

NERVE
B 6. This potential would exist if the cell membrane suddenly become selectively and completely permeable only to that ion :
A. membrane potential C. action potential
B. equilibrium potential D. generator potential
* Textbook of Medical Physiology 10th ed, by Guyton and Hall

C 7. The concentration of acetylcholine in the synaptic cleft is determined by:


A. the rate of active uptake of the transmitter by the surrounding neurons
B. the amount of transmitter released by the presynaptic nerve terminal
C. the rate of enzymatic breakdown of the transmitter in the synaptic cleft
D. the diffusion rate of the transmitter from the presynaptic nerve terminal to the synaptic cleft
* Textbook of Medical Physiology 10th ed, by Guyton and Hall

C 8. Which of the following statements is TRUE regarding the propagation of an action potential along a nerve axon?
A. the propagation of action potentials requires a direct source of ATP
B. multiple sclerosis is associated with increased conduction velocity
C. large myelinated nerve fibers conduct faster than small unmyelinated nerve fibers
D. saltatory conduction is associated with a slowing of action potential propagation
* Textbook of Medical Physiology 10th ed, by Guyton and Hall

C 9. Ion flux through this channel mediates the repolarization phase of the neuronal action potential:
A. calcium channels C. potassium channels
B. sodium channels D. ligand-gated channels
* Textbook of Medical Physiology 10th ed, by Guyton and Hall

A 10. The absolute refractory period:


A. results from the inactivation of the voltage-gated Na+ channels which cannot be opened until the cell is repolarized
B. represents period of hyperpolarization
C. is the time during which stronger than normal stimuli are required to cause excitation
D. is mediated by the slow closure of the voltage-gated K+ channels
* Textbook of Medical Physiology 10th ed, by Guyton and Hall

B 11. The following characterize an action potential EXCEPT:


A. involves opening and closure of voltage-gated Na+- K+ channels
B. is a graded response
C. usually brief in duration, about 1-10msec.
D. Occurs in spike form, an all-or-none response
* Textbook of Medical Physiology 10th ed, by Guyton and Hall

B 12. What would be the effect of changing the RMP from –70mV to –50mV?
A. threshold would not be reached
Physiology
20 / 50

B. a weaker stimulus would be required to reach threshold


C. the magnitude of the action potential would increase
D. the stimulus required to reach threshold would not change
* Textbook of Medical Physiology 10th ed, by Guyton and Hall

MUSCLE
B 13. Which of the following DOES NOT describe fast twitch muscles?
A. powerful contraction C. rapid accumulation of lactate
B. abundant mitochondria D. anaerobic ATP production
* Review of Medical Physiology 20th ed, by Ganong

A 14. The chief cause of rigor mortis is:


A. decline in ATP production
B. decrease in muscle Ca2+
C. release of lysosomal enzymes in the muscle
D. death of neuromuscular junctions
* Review of Medical Physiology 20th ed, by Ganong

D 15. NOT a change in the muscular tissue of a highly trained distance runner:
A. increase in myoglobin content
B. increase in mitochondria
C. hypertrophy of the individual fibers
D. increase in number of muscle fibers
* Review of Medical Physiology 20th ed, by Ganong

B 16. This ATP-requiring system pumps Ca2+ from the environment of myofibrils back into the calsequestrin-rich compartment:
A. transverse tubule C. sarcomere
B. sarcoplasmic reticulum D. muscle cell membrane
* Review of Medical Physiology 20th ed, by Ganong

A 17. Which of the following events immediately precedes depolarization of the transverse tubules in skeletal muscles?
A. depolarization of sarcolemmal membrane
B. sequestration of Ca2+ by the Ca2+-ATPase
C. opening of the Ca2+ -release channels on sarcoplasmic reticulum
D. actin and myosin binding
* Review of Medical Physiology 20th ed, by Ganong

A 18. The counterpart of calmodulin in skeletal muscles:


A. troponin B. tropomyosin C. actin D. calsequestrin
* Textbook of Medical Physiology 10th ed, by Guyton and Hall

A 19. Ion flux through this channel is most closely associated with the upstroke of action potentials and contraction in smooth muscle cells:
A. calcium channels C. potassium channels
B. sodium channels D. ligand-gated channels
* Textbook of Medical Physiology 10th ed, by Guyton and Hall

A 20. Slow twitch fibers :


A. contain large amounts of red pigment myoglobin
B. can't sustain contraction for long periods of time
C. have fewer capillaries and mitochondria
D. have lesser amounts of enzyme myosin ATPase
* Textbook of Medical Physiology 10th ed, by Guyton and Hall

A 21. In skeletal muscle contraction:


A. ATP is used to cause the myosin cross-bridge to release from the actin molecule, leading to muscle relaxation
B. Calcium combines with actin to expose an actin binding site for combination with myosin
C. Calcium combines with calmodulin to activate myosin light chain kinase
D. the myosin filament shortens to reduce the length of the sarcomere
* Textbook of Medical Physiology 10th ed, by Guyton and Hall

D 22. Calcium:
A. initiates contraction of striated muscle by binding to regulatory proteins on the thick filament
B. initiates contraction of smooth muscle by direct binding to the thin filament
C. binds to calmodulin to initiate contraction of striated muscle
D. binds to calmodulin to initiate contraction of smooth muscle
* Textbook of Medical Physiology 10th ed, by Guyton and Hall

BLOOD
C 23. Cytoplasmic enzymes in red blood cells are capable of forming NADPH, which in turn serves the RBC by:
A. preventing oxidation of glucose
B. keeping the iron of the cell's hemoglobin in the ferric form
C. maintaining membrane transport of ions
D. A and B only
* Textbook of Medical Physiology 10th ed, by Guyton and Hall

Physiology
21 / 50

A 24. The process by which neutrophils and monocytes squeeze through the pores of the blood vessels is:
A. diapedesis C. chemotaxis
B. pavementing D. opsonization
* Textbook of Medical Physiology 10th ed, by Guyton and Hall

B 25. When coumarin , such as warfarin, is given to a patient, the plasma level of which of thefollowing clotting factors will decrease?
A. fibrinogen C. Hageman factor
B.prothrombin D. anti-hemophilic factor A
* Textbook of Medical Physiology 10th ed, by Guyton and Hall.

AUTONOMICS
D 26. The difference between the sympathetic nervous system and the parasympathetic nervous system is that in the latter:
A. the peripheral ganglia are located near the center
B. acetylcholine and norepinephrine are utilized as neurotransmitter agents at the neuroeffector junction
C. the effects are voluntary
D. the effects are more localized
* Textbook of Medical Physiology 10th ed, by Guyton and Hall.

C 27. Sympathetic dysfunction will cause:


A. vasoconstriction C. pupillary constriction
B. tachycardia D. constipation
* Textbook of Medical Physiology 10th ed, by Guyton and Hall.

A 28. If a beta-1 blocker is given to a patient, which of the following will decrease?
A. heart rate C. pupillary size
B. intestinal motility D. bronchial airway diameter
* Textbook of Medical Physiology 10th ed, by Guyton and Hall.

B 29. Adrenergic receptors in smooth muscle and glands are:


A. alpha 1 and beta 1 C. alpha 2 and beta 1
B. alpha 1 and beta 2 D. alpha 2 and beta 2
* Textbook of Medical Physiology 10th ed, by Guyton and Hall.

B 31. The main mechanism that will deactivate acetylcholine is by:


A. methylation by acetyltransferase
B. hydrolysis by acetylcholinesterase
C. diffusion away from the neuroeffector junction
D. reuptake by the pre-junctional fiber
* Textbook of Medical Physiology 10th ed, by Guyton and Hall

CARDIOVASCULAR
B 32. The myocardial cells for contraction :
A. are automatic cells
B. have intercalated discs
C. contain less mitochondria than skeletal muscles
D. have fewer and unbranched T-tubules
* Textbook of Medical Physiology 10th ed, by Guyton and Hall

B 33. Action potential of the SA node:


A. has a distinct and rapid phase 0
B. has slow phase of depolarization
C. is characterized with a threshold voltage of -90mV
D. has constant phase 4
* Textbook of Medical Physiology 10th ed, by Guyton and Hall

D 34. AV conduction delay is due to:


A. increase in the frequency of AP discharge of the SA node
B. sympathetic stimulation
C. longer refractory period
D. few gap junctions connecting the muscle cells
* Textbook of Medical Physiology 10th ed, by Guyton and Hall

B 35. Contractility of myocardial muscles:


A. is only due to the influence of intracellular Ca2+
B. manifests all-or-none response
C. starts immediately after the end of the action potential
D. is not affected by autonomic nervous system
* Textbook of Medical Physiology 10th ed, by Guyton and Hall

B 36. A decrease in arterial distensibility will:


A. increase venous compliance
B. increase pulse pressure
Physiology
22 / 50

C. decrease afterload
D. decrease end systolic volume
* Textbook of Medical Physiology 10th ed, by Guyton and Hall

A 37. Extrinsic control of the resistance vessels depend on:


A. norepinephrine acting on alpha receptors
B. parasympathetic stimulation
C. somatic nervous system stimulation
D. dilator response of the splanchnic circulation during sympathetic activity
* Textbook of Medical Physiology 10th ed, by Guyton and Hall

D 38. The capillary hemodynamics:


A. favor filtration than reabsorption
B. is affected more by electrolyte concentration than plasma proteins
C. are dependent on same physical properties as that of the arteries and veins
D. is determined primarily by an interplay between capillary hydrostatic pressure and plasma oncotic pressure
* Textbook of Medical Physiology 10th ed, by Guyton and Hall

A 39. The most important function of the microcirculation is:


A. exchange of nutrients and wastes between blood and tissue
B. filtration of water through capillaries
C. Regulation of vascular resistance
D. autoregulation of blood flow
* Textbook of Medical Physiology 10th ed, by Guyton and Hall

B 40. In a normal cardiac cycle:


A. the first heart sound coincides with the beginning of ventricular ejection
B. the second heart sound coincides with the end of ventricular ejection
C. the highest left ventricular pressure is reached just as the aortic valve closes
D. atrial systole occurs during ventricular isovolumetric relaxation
* Textbook of Medical Physiology 10th ed, by Guyton and Hall

B 41. Which of the following statements about the determinants of blood pressure is FALSE?
A. pulse pressure is primarily determined by stroke volume and aortic compliance
B. an increase in heart rate will change mean arterial pressure if cardiac output remains constant
C. mean arterial pressure is determined solely by cardiac output and systemic vascular resistance
D. if stroke volume increases and cardiac output remains constant, diastolic pressure decreases
* Textbook of Medical Physiology 10th ed, by Guyton and Hall

RESPIRATORY PHYSIOLOGY
A 42. Which of the following is higher at the apex of the lung than at the base when a person is standing?
A. V/Q ratio C. Perfusion
B. Ventilation D. Compliance
* Review of Medical Physiology 20th ed, by Ganong

C 43. This is increased during moderate aerobic exercise;


A. lactic acid C. ventilation
B. carbon dioxide D. hydrogen ion
* Review of Medical Physiology 20th ed, by Ganong

A 44. Reduced arterial PO2 is observed in;


A. hypoxic hypoxia C. anemic hypoxia
B. stagnant hypoxia D. histotoxic hypoxia
* Review of Medical Physiology 20th ed, by Ganong

C 45. The factor that primarily affects the diffusing capacity of the lungs for CO 2;
A. alveolar surface area C. diffusion coefficient
B. thickness of the membrane D. CO2 partial pressure gradient
* Review of Medical Physiology 20th ed, by Ganong

C 46. Which of the following factors favors the uptake of oxygen in the lungs?
A. hyperthermia C. alkalosis
B. exercise D. hypercapnea
* Review of Medical Physiology 20th ed, by Ganong

D 47. These receptors are primarily stimulated by increasing lung inflation;


A. peripheral chemoreceptors C. central chemoreceptors
B. J receptors D. pulmonary stretch receptors
* Physiology by Berne 5th edition

A 48. Rhythmic breathing depends on a continuous inspiratory drive from the;


A. dorsal inspiratory group C. pontine centers
B. vagal afferent fibers D. ventral respiratory group
* Physiology by Berne 5th edition

Physiology
23 / 50

A 49. The decrease in O2 affinity of hemoglobin with high concentration of CO2 is called;
A. Bohr effect C. Haldane effect
B. Cl- shift D. physiologic shunting
* Review of Medical Physiology 20th ed, by Ganong

D 50. Arterial PO2 is increased in;


A. anatomic shunt C. physiologic shunt
B. low V/Q ratio D. hyperventilation
* Physiology by Berne 5th edition

C 51. Which of the following is increased during inspiration?


A. intrapleural pressure C. intrathoracic volume
B. outflow of air D. intraalveolar pressure
* Review of Medical Physiology 20th ed, by Ganong

RENAL PHYSIOLOGY
D 52. Which of the following would tend to increase glomerular filtration rate?
A. an increase in glomerular capillary oncotic pressure
B. vasoconstriction of the afferent arteriole
C. an increase in hydrostatic pressure in Bowman's capsule
D. an increase in renal blood flow
* Textbook of Medical Physiology 10th ed, by Guyton and Hall.

C 53. If a substance has a transport maximum ( Tm) for reabsorption, this means:
A. reabsorption is only passive
B. only a constant fraction of the substance will be reabsorbed
C. below a threshold level, all of the substance will be reabsorbed
D. phlorizin blocks reabsorption
* Textbook of Medical Physiology 10th ed, by Guyton and Hall.

D 54. Which of the following substances has the lowest renal clearance?
A. inulin B. urea C. creatinine D. glucose
* Textbook of Medical Physiology 10th ed, by Guyton and Hall.

C 55. Formation of concentrated urine is:


A. dependent on the collecting duct being impermeable to water
B. associated with water diuresis
C. dependent on the hypertonicity of the medullary interstitial fluid
D. dependent on the isotonicity of the cortical interstitial fluid
* Textbook of Medical Physiology 10th ed, by Guyton and Hall.

A 56. Antidiuretic hormone (vasopressin):


A. renders the distal renal tubule permeable to water C. increases urine output
B. enhances free water clearance D. decreases urine tonicity
* Textbook of Medical Physiology 10th ed, by Guyton and Hall.

D 57. Aldosterone:
A. diminishes reabsorption of sodium in the distal renal tubule
B. diminishes the secretion of potassium in exchange for sodium
C. eventually increases ECFV
D. eventually increases arterial blood pressure
* Textbook of Medical Physiology 10th ed, by Guyton and Hall.

B 58. Renin:
A. converts angiotensin I to angiotensin II
B. is secreted by the juxtaglomerular cells of the kidneys
C. is secreted in response to afferent arteriolar vasodilatation
D. is a peptide
* Textbook of Medical Physiology 10th ed, by Guyton and Hall.

C 59. In the kidneys, glucose is reabsorbed in the proximal convoluted tubule co-transported with:
A. potassium C. sodium
B. hydrogen D. chloride
* Textbook of Medical Physiology 10th ed, by Guyton and Hall.

B 60. Elevated bicarbonate concentration with normal arterial pCO2 gives rise to:
A. metabolic acidosis C. respiratory acidosis
B. metabolic alkalosis D. respiratory alkalosis
* Textbook of Medical Physiology 10th ed, by Guyton and Hall.

Physiology
24 / 50

A 61. In kidney shutdown, the plasma concentration of this substance is decreased:


A. bicarbonate C. potassium
B. hydrogen D. sodium
* Textbook of Medical Physiology 10th ed, by Guyton and Hall.

GIT
D 62. Which of the following statements is INCORRECT about the peristaltic movement of the GIT?
A. it is the basic propulsive movement of the GIT
B. the usual stimulus is distention of the intestine
C. it will move GI contents analward
D. effective peristalsis requires an active Meissner's plexus
* Textbook of Medical Physiology 10th ed, by Guyton and Hall.

D 63. The more potent signals controlling rate of gastric emptying are coming from the:
A. esophagus C. pyloric region
B. body of the stomach D. duodenum
* Textbook of Medical Physiology 10th ed, by Guyton and Hall.

B 64. Mass movements of the colon are initiated by:


A. haustral contractions
B. intense stimulation of the parasympathetics
C. enterogastric reflexes
D. intense stimulation of the sympathetics
* Textbook of Medical Physiology 10th ed, by Guyton and Hall.

A 65. The secretory activities of the salivary glands are:


A. controlled mainly by the parasympathetic neurons
B. not affected by reflexes originating from the stomach and duodenum
C. inhibited by sympathetics from the superior cervical region
D. independent of its blood supply
* Textbook of Medical Physiology 10th ed, by Guyton and Hall.

A 66. Pancreatic digestive enzymes are NOT capable of hydrolyzing:


A. starches into monosaccharides
B. neutral fats into fatty acids
C. peptides into amino acids
D. cholesterol esters
* Textbook of Medical Physiology 10th ed, by Guyton and Hall

MOTOR SYSTEM
B 67. The ability of four-legged animals to effectively support the body against gravity without any signals from the cortex is dependent on the activity of
the:
A. mesencephalon C. medullary reticular nuclei
B. pontine reticular nuclei D. striate body
* Textbook of Medical Physiology 10th ed, by Guyton and Hall.

D 68. Which of the following is NOT a motor function of the cerebellum?


A. it helps control intensity of muscle contraction
B. it is essential for the timing of motor activities
C. it is needed for rapid progression from one movement to the next
D. it helps in planning and controlling complex patterns of movement
* Textbook of Medical Physiology 10th ed, by Guyton and Hall.

HIGHER CENTERS
C 69. Sensory impulses from thius sensory organs/tissues do not pass through the thalamus:
A. gustatory organs C. olfactory organs
B. nociceptive tissues D. proprioceptive tissues
* Textbook of Medical Physiology 10th ed, by Guyton and Hall.

C 70. Essential to transfer thoughts, memories, training and other information between the two cerebral hemispheres:
A. hippocampus C. anterior commissure
B. thalamus D. arcuate fasciculus
* Textbook of Medical Physiology 10th ed, by Guyton and Hall.

GENERAL SENSES
A 71. The following tactile receptors are mostly involved in detection of vibration:
A. Pacinian and Meisnner's corpuscles
B. electromagnetic receptors
C. Krause and Bulb
D. Muscle spindle and Golgi tendon organs
* Textbook of Medical Physiology 10th ed, by Guyton and Hall.
Physiology
25 / 50

C 72. This type of sensation is transmitted in the dorsal column pathway:


A. pain sensation C. position sensation
B. thermal sensation D. tickle and itch
* Textbook of Medical Physiology 10th ed, by Guyton and Hall.

VISION
A 73. The blind spot cause by lack of rods and cones in the retina is:
A. found about 15 degrees lateral to the central point of vision
B. in the optic chiasm
C. in the fovea centralis
D. found about 15 degrees medial to the central point of vision
* Textbook of Medical Physiology 10th ed, by Guyton and Hall.

C 74. Control of pupillary diameter directly involves:


A. contraction or relaxation of the ciliary muscles
B. increase or decrease in the strength of the lens
C. contraction or relaxation of the muscles of the iris
D. increase or decrease tension in the suspensory ligaments
* Textbook of Medical Physiology 10th ed, by Guyton and Hall.

HEARING
C 75. This mechanism is NOT used to determine direction from which a sound emanates:
A. time lag between the entry of sound into one ear and into the opposite ear
B. difference between intensities of the sounds in the two ears
C. difference between pitches of the sounds in the two ears
D. difference between the qualities of sound or timbre
* Textbook of Medical Physiology 10th ed, by Guyton and Hall.

D 76. A person is completely deaf if the:


A. tympanic membrane is destroyed
B. the human ear is not capable of conducting sound waves through air
C. the auditory ossicles are ankylosed
D. cochlea is destroyed
* Textbook of Medical Physiology 10th ed, by Guyton and Hall.

TASTE AND SMELL


D 77. The taste receptors are more sensitive to this type of taste than all the other types:
A. sweet C. sour
B. salty D. bitter
* Textbook of Medical Physiology 10th ed, by Guyton and Hall.

B 78. Which is CORRECT regarding sense of smell?


A. Humans have more developed sense of smell than animals
B. receptors are actually bipolar neurons derived from central neurons
C. the olfactory membrane has a surface area of about 4.2cm2
D. it is easily studied compared to other senses
* Textbook of Medical Physiology 10th ed, by Guyton and Hall.

PITUITARY GLAND
C 79. Secretion of the growth hormone is increased by:
A. increased blood glucose C. starvation
B. increased somatomedins D. B and C only
* Textbook of Medical Physiology 10th ed, by Guyton and Hall.

A 80. Which of the following is NOT present in SIADH?


A. hypernatremia C. increased blood volume
B. decreased urine output D. increased blood pressure
* Review of Medical Physiology 20th ed, by Ganong

THYROID GLAND
B 81. The most immediate effect of TSH administration on the thyroid glands is to increase:
A. the activity of the iodide pump
B. proteolysis of the thyroglobulin
C. oxidation of iodides to iodine
D. growth of the thyroid follicular cells
* Textbook of Medical Physiology 10th ed, by Guyton and Hall.

C 82. Which of the following is NOT a clinical manifestation of hypothyroidism?


A. weight gain C. pretibial myxedema
B. bradycardia D. cold intolerance
* Review of Medical Physiology 20th ed, by Ganong
Physiology
26 / 50

PARATHYROID GLANDS
D 83. PTH and calcitonin are physiologic antagonists. However, both hormones can:
A. increase bone resorption
B. increase intestinal absorption of calcium
C. decrease the serum calcium level
D. decrease the serum phosphate level
* Textbook of Medical Physiology 10th ed, by Guyton and Hall.

C 84. One difference between the actions of PTH and Vit. D is that the latter decreases:
A. bone resorption C. PO4 excretion
B. calcium excretion D. serum PO4 level
* Textbook of Medical Physiology 10th ed, by Guyton and Hall.

ISLETS
C 85. In insulin deficiency, the activity of this/these enzymes will increase:
A. glycogen synthetase C. glucose pho\sphatase
B. phosphorylase D. A and B only
* Textbook of Medical Physiology 10th ed, by Guyton and Hall.

A 86. In the liver, glucagon increases:


A. protein catabolism C. glycolysis
B. lipogenesis D. glycogenesis
* Textbook of Medical Physiology 10th ed, by Guyton and Hall.

ADRENALS
D 87. The diabetogenic effects of cortisol include:
A. increased gluconeogenesis
B. decrease peripheral utilization of glucose
C. increased glycogenolysis in the liver
D. A and B only
* Textbook of Medical Physiology 10th ed, by Guyton and Hall.

C 88. Which of the following is NOT present in primary hyperaldosteronism?


A. hypertension C. hyperkalemia
B. muscle paralysis D. increased ECF volume
* Textbook of Medical Physiology 10th ed, by Guyton and Hall.

MALE GONADS
D 89. In males, FSH stimulates:
A. the sertoli cells to secrete inhibin
B. spermatogenesis
C. the interstitial cells of Leydig to secrete testosterone
D. A and B only
* Textbook of Medical Physiology 10th ed, by Guyton and Hall.

B 90. Which of the following is NOT an effect of testosterone in the adult male?
A. increased thickness of the skin
B. increased growth of hair on the scalp
C. increased erythropoiesis
D. increased sodium reabsorption in the distal tubules of the kidneys
* Textbook of Medical Physiology 10th ed, by Guyton and Hall.

FEMALE GONADS
C 91 Which of the following is NOT TRUE about the pre-ovulatory phase of the female monthly cycle?
A. a slow and moderate increase in estrogen secretion will have a negative feedback effect on FSH and LH
B. FSH initiates follicular growth
C. progesterone stimulates secretory activity of the endometrial glands
D. LH brings about ovulation
* Textbook of Medical Physiology 10th ed, by Guyton and Hall.

B 92 Which of the following is NOT an effect of estrogen?


A. stimulates stromal and ductal growth of the breasts
B. stimulates production of a cervical mucus that is scanty and viscous
C. stimulates growth of the ovarian follicles
D. stimulates osteoblastic activity
* Textbook of Medical Physiology 10th ed, by Guyton and Hall.

Physiology
27 / 50

AVIATION PHYSIOLOGY
C 93 Which one of the following does not a explain the events seen in chronic mountain sickness?
A. Red cell mass becomes so great that the blood viscosity increases several fold
B. Alveolar arteriolar spasm diverts much of the blood flow through nonalveolar pulmonary vessels
C. increased viscosity increases tissue blood flow
D. pulmonary arterioles become exceptionally vasospastic because of the lung hypoxia
* Textbook of Medical Physiology 10th ed, by Guyton and Hall.

B 94 Which one of the following is not an important acute effect of hypoxia?


A. coma and death
B. Hypertension, dyspnea and headache
C. twitchings and seizures
D. drowsiness, lassitude, mental and muscle fatigue
* Textbook of Medical Physiology 10th ed, by Guyton and Hall.

DEEP SEA PHYSIO


B 95 It has been learned that hyperbaric oxygen therapy has valuable therapeutic effects.
In which condition will this therapy benefit the patients most?
A. Diabetic foot ulcers
B. infection with clostridial organisms
C. Carbon dioxide poisoning
D. leptospirosis
* Textbook of Medical Physiology 10th ed, by Guyton

D 96 Signs and symptoms of decompression sickness is best explained by which of the following mechanism?
A. gas bubbles accumulating in the muscles causing muscle cramps
B. decrease in the oxygen carrying capacity of red blood cells
C. accumulation of carbon dioxide in different tissues of the body
D. gas bubbles blocking many blood vessels in the different tissues
* Textbook of Medical Physiology 10th ed, by Guyton

BODY TEMPERATURE REGULATION


C 97 The removal of heat from the body by air currents after the heat has been transferred to the air is called?
A. Radiation
B. conduction
C. convection
D. evaporation
* Textbook of Medical Physiology 10th ed, by Guyton

A 98 The following are factors that determine the rate of heat production, called the metabolic rate of the body. Which one of the following is
not a factor of this?
A. rate of heat lost from the skin to the surroundings
B. basal rate of metabolism of all the cells of the body
C. extra rate of metabolism caused by muscle activity
D. extra metabolism caused by increased chemical activity in the cells
* Textbook of Medical Physiology 10th ed, by Guyton

SPORTS PHYSIO
B 99 It is the immediate source of energy actually used for muscle contraction and is sufficient to
sustain maximal power for only about 3 secs?
A. Phosphocreatine-creatine system
B. Adenosine triphosphate
C. Glycogen-lactic acid system
D. Aerobic system
* Textbook of Medical Physiology 10th ed, by Guyton

Physiology
28 / 50

A11100 Which one of the following explain the changes that occur in a hypertrophied muscle fibers?
A. Up to a 120 percent increase in mitochondrial enzymes
B. decrease in number of myofibrils
C. as much as 50 percent decrease in stored glycogen
D. as much as a 60 - 80 percent decrease in the components of phosphagen system
* Textbook of Medical Physiology 10th ed, by Guyton and Hall.

In nerve cell membranes, a change in transmembrane voltage triggers opening of sodium channels, which in turn, causes greater change in
transmembrane voltage. This system operates in a/an:
A. open loop mechanism C. servo mechanism
B. positive feedback mechanism d. D. negative feedback mechanism
(Answer: B/ Reference: Guyton 10th ed, Ch 4, p 59/ MPL 90)

One way of enhancing neuromuscular transmission is by:


A. decreasing the release of acetylcholine.
B. increasing the destruction of acetylcholine.
C. increasing the concentration of calcium ions.
D. decreasing the concentration of sodium ions.
(Answer: C/ Reference: Guyton 10th ed, Ch 7, p 82 - 83/ MPL 60)

During the absolute refractory period of the nerve action potential, the axon:
A. shows zero excitability.
B. shows decreased threshold on stimulation.
C. shows increased permeability to chloride ions.
D. can be stimulated with supramaximal stimulus.
(Answer: A/ Reference: Guyton 10th ed, Ch 5, p 64/ MPL 60)

In multiple sclerosis, there is progressive degeneration of the myelin sheath of CNS neurons. The neurologic manifestations of this disease
include:
A. sensory but no motor deficit C. both sensory & motor deficits
B. motor but no sensory deficit D. neither sensory nor motor deficit
(Answer: A/ Reference: Guyton 9th ed, Ch 5, p 68/ MPL 33)

The initial segment of the axon is functionally important because this is the zone where:
A. saltatory conduction takes place.
B. integration of nerve signals occur.
C. action potentials are normally generated.
D. laying down myelin sheath normally begins.
(Answer: C/ Reference: Ganong 20th ed, Ch 2, p 53/ MPL 60)

Which of the following is TRUE regarding deactivation of acetylcholine?


A. The by-products of enzymatic breakdown of acetylcholine are choline &
acetate.
B. Acetylcholine that has combined with post-synaptic receptors cannot be
degraded.
C. Acetylcholinesterase inhibitors act primarily by decreasing acetylcholine
release from the presynaptic cell.
D. Uptake of acetylcholine into the presynaptic cell is the most important
mechanism in terminating acetylcholine signal.
(Answer: A/ Reference: Guyton 10th ed, Ch 7, p 81-82/ MPL 33)

The type of inhibition that results from decreased transmitter release by the presynaptic cell in response to an action potential:
A. direct inhibition C. feedback inhibition
B. presynaptic inhibition D. feedforward inhibition
(Answer: B/ Reference: Ganong 20th ed, Ch 4, p 93/ MPL 60)

Which of the following local potentials are hyperpolarizing?


A. end plate potentials C. miniature end plate potentials
B. inhibitory post synaptic potentials D. excitatory post synaptic potentials
(Answer: B/ Reference: Guyton 10th ed, Ch 45, p 523/ MPL 90)

Myasthenia gravis is a clinical condition characterized by destruction of neuromuscular receptors. The muscle weakness in myasthenia gravis may
improve by giving a drug that prevents the:
A. release of acetylcholine C. diffusion of acetylcholine
B. synthesis of acetylcholine D. hydrolysis of acetylcholine
(Answer: D/ Reference: Guyton 10th ed, Ch 7, p 83/ MPL 90)

There is an inverse relationship between the velocity of muscle contraction & the:
A. intensity of the stimulus.
B. length of the sarcomere.

Physiology
29 / 50

C. load on the muscle during contraction.


D. number of cross bridges formed during contraction.
(Answer: C/ Reference: Guyton 10th ed, Ch 6, p 73/ MPL 60)

According to the law of adequate stimulus, sensory receptors:


A. respond to only one type of stimulus.
B. are nerve endings of sensory neurons.
C. are located close to the site of stimulation.
D. have low threshold to only one form of stimulation.
(Answer: D/ Reference: Ganong 20th ed, Ch 5, p 122/ MPL 33)

Which of the following statements best describes adequate stimulus?


A. It is the only form of stimulus, which can activate a sensory receptor.
B. It is the minimum stimulus intensity that activates a sensory receptor.
C. It is the form of energy to which a particular receptor has lowest threshold.
D. It is the specific form of stimulus that can generate an action potential in the sensory nerve.
(Answer: C/ Reference: Ganong 20th ed, Ch 5, p 122/ MPL 25)

The sympathetic & parasympathetic nervous systems are considered physiologic antagonists at the:
A. sweat glands C. lacrimal glands
B. adrenal medulla D. intestinal smooth muscles
th
(Answer: D/ Reference: Guyton 10 ed, Ch 60, p 702/ MPL 60)

One of the following conditions is due to a serious derangement of temperature regulator mechanism:
A. heat stroke C. heat cramps
B. hyperpyrexia D. heat exhaustion
(Answer: A/ Reference: Guyton 10th ed, Ch 73, p 831/ MPL 60)

A subject focuses his eyes on an object. The image of the object will fall on the:
A. lens C. optic disc
B. ora serrata D. fovea centralis
th
(Answer: D/ Reference: Ganong 20 ed, Ch 8, p 152/ MPL 60)

The error of refraction characterized by irregular curvature of the cornea:


A..myopia C. cataract
B..presbyopia D. astigmatism
(Answer: D/ Reference: Guyton 10th ed, Ch 49, p 572/ MPL 90)

A 35-year old female patient was tested for visual acuity using the Snellen's chart. The result was 20/100. This means that the subject has:
A. farsightedness C. retinal disorder
B. needs bifocal lenses D. poorer than normal vision
th
(Answer: D/ Reference: Guyton 10 ed, Ch 49, p 574/ MPL 60)

In a normal subject, infusion of warm water into the left ear results in:
A. deviation of eyeballs to the left. C. upward deviation of eyeballs.
B. deviation of eyeballs to the right. D. bitemporal deviation of eyeballs.
(Answer: A/ Reference: Ganong 20th ed, Ch 9, p 187/ MPL 60)

A 50-year old patient diagnosed with unilateral nerve deafness, as tested by Weber method, would present with the following:
A. sound louder in the normal ear
B. bone conduction better than normal
C. bone conduction worse than normal
D. air conduction noted after bone conduction is over
(Answer: A/ Reference: Ganong 20th ed, Ch 9, p 185/ MPL 33)

A 35-year old patient suffers from complete memory loss for events that occurred after a blow to the head. This condition is termed:
A. global amnesia C. sporadic amnesia
B. retrograde amnesia D. anterograde amnesia
(Answer: B/ Reference: Ganong 20th ed, Ch 16, p 273/ MPL 90)
A subject who talks quite fluently was asked to identify a triangle drawn on a cardboard but was unable to identify it even if he follows the card with
his eyes wherever it is moved. The patient most likely has a lesion in the:
A. retina C. optic chiasma
B. primary visual cortex D. visual association cortex
(Answer: D/ Reference: Guyton 10th ed, Ch 51, p 1893-4/ MPL 60)

Which of the following phenomena occurs during NREM sleep?


A. increased muscle tone C. increased blood pressure
B. conjugate eye movement D. increased growth hormone
(Answer: A/ Reference: Guyton 10th ed, Ch 59, p 659-60/ MPL 33)

A person wears a perfume but after sometime, he fails to perceive the smell of the perfume. This sensory phenomenon is:
A. fatigue C. contrast
B. adaptation D. afterdischarge
(Answer: B/ Reference: Guyton 10th ed, Ch 53, p 617/ MPL 90)

Physiology
30 / 50

The anemia due to maturation failure of the RBC:


A. hemolytic anemia C. microcytic anemia
B. pernicious anemia D. normocytic anemia
(Answer: B/ Reference: Guyton 10th ed, Ch 32, p 386/ MPL 90)

Which of the following is a result of acclimatization to high altitude?


A. increased hematocrit C. increased platelet count
B. increased prothrombin time D. increased reticulocyte count
(Answer: A/ Reference: Guyton 10th ed, Ch 32, p 390/ MPL 90)

A 6-year old child was brought to the clinic for crampy abdominal pain. Stool exam showed 1 -2 Ascaris ova/high power field. Which of the
following blood cells would be significantly elevated on complete blood count?
A. basophils C. monocytes
B. eosinophils D. lymphocytes
th
(Answer: B/ Reference: Guyton 10 ed, Ch 33, p 399/ MPL 90)

In the isovolumetric phase of systole:


A. AV valves are open.
B. The 2nd heart sound is produced.
C. Ventricular volume rapidly increases.
D. Ventricular pressure rapidly increases.
(Answer: D/ Reference: Guyton 10th ed, Ch 9, p 100/ MPL 60)

Increased sympathetic discharge during exercise will LEAST likely result in:
A. negative inotropic effect C. faster impulse conduction
B. positive chronotropic effect D. enhanced myocardial contractility
(Answer: A/ Reference: Guyton 10th ed, Ch 29, p 574/ MPL 60)

Which of the following parameters is the best estimate of ventricular preload?


A. arterial blood pressure C. myocardial contractility
B. ventricular end-diastolic pressure D. ventricular end-systolic pressure
th
(Answer: B/ Reference: Guyton 10 ed, Ch 9, p 103/ MPL 60)

The resting membrane potential of the ventricular muscle is predominantly determined by the selective permeability of the membrane to:
A. sodium C. calcium
B. potassium D. chloride
(Answer: B/ Reference: Guyton 10th ed, Ch 3, p 78/ MPL 60)

One of the following statements regarding the cardiac cycle is CORRECT:


A. It spans the whole of ventricular systole.
B. The electrical events are preceded by the mechanical events.
C. One third of the stroke volume is ejected during rapid ejection phase.
Ventricular repolarization occurs slightly prior to the end of ventricular
contraction.
(Answer: B/ Reference: Guyton 10th ed, Ch 9, p 100/ MPL 60)

Which of the following cardiac pacemakers conduct faster than any other tissue in the heart?
A. Sinoatrial node C. Bundle of His
B. Atrioventricular node D. His - Purkinje system
(Answer: A/ Reference: Guyton 10th ed, Ch 10, p 111/ MPL 90)

The physiologic significance of the AV nodal delay is to allow sufficient time for:
A. Atrial relaxation C. Filling of the ventricles
B. Venous return to the atria D. Contraction of the ventricles
th
(Answer: C/ Reference: Guyton 10 ed, Ch 10, p 109/ MPL 60)

The 1st heart sound is caused primarily by the:


A. Opening of the mitral valve
B. Opening of the semilunar valves
C. Closure of the AV valves and related vibrations
D. Surging of blood toward the AV valves to initiate closure
(Answer: C/ Reference: Ganong 20th ed, Ch 29, p 571/ MPL 90)

The stroke volume increases during exercise due to an increase in:


A. Heart rate C. Venous return
B. Peripheral resistance D. Circulating blood volume
(Answer: C/ Reference: Guyton 10th ed, p 213/ MPL 33)

When a person assumes an upright position from supine, which of the following compensatory changes occur?
A. Decreased heart rate
B. Decreased cardiac output
C. Increased cardiac contractility
D. Decreased total peripheral resistance
(Answer: B/ Reference: Ganong 20th ed, Ch 29, p 574/ MPL 33)

Physiology
31 / 50

Turbulent blood flow is least likely to occur in:


A. Anemia C. Increased vessel radius
B. Increased blood viscosity D. Increased velocity of blood flow
(Answer: B/ Reference: Ganong 20th ed, Ch 30, p 585/ MPL 33)

Which of the following is true of the Riva Rocci method of blood pressure determination?
A. It makes use of the Korotkoff sounds.
B. It is a direct means of BP measurement.
C. Only the diastolic pressure is determined.
D. It avoids errors caused by the auscultatory gap.
(Answer: A/ Reference: Ganong 20th ed, Ch 30, p 591-2/ MPL 90)
A patient is seen with symptoms of anterior wall myocardial infarction. Arterial blood pressure is 70/40 mm Hg and cardiac output is only 1/3 of
normal. Which of the following would be expected of the central venous pressure?
A. Increased
B. Not changed
C. Decreased because of decreased heart rate
D. Decreased because of decreased venous return
(Answer: D/ Reference: Guyton 10th ed, Ch 15, p 157/ MPL 60)

The volume of blood left in the ventricle after a maximal ventricular contraction:
A. Residual volume C. End-diastolic volume
B. End-systolic volume D. Systolic reserve volume
(Answer: B/ Reference: Ganong 20th ed, Ch 29, p 568/ MPL 90)

The velocity of blood flow is increased with an increase in:


A. Resistance
B. Vessel length
C. Blood velocity
D. Pressure gradient between vessel ends
(Answer: D/ Reference: Guyton 10th ed, Ch 14, p 150/ MPL 33)

Which of the following best describes the auscultatory gap?


A. Disappearance of Korotkoff sounds at the level of the diastolic pressure
B. Disappearance of Korotkoff sounds below the level of the diastolic pressure
C. Disappearance of Korotkoff sounds at the level above the systolic pressure
D. Disappearance of Korotkoff sounds at the level above the diastolic pressure
(Answer: D/ Reference: Ganong 20th ed, p 592/ MPL 60)

Within physiologic limits, an increase in right atrial pressure will:


A. Decrease intrathoracic pressure C. Decrease heart rate
B. Decrease systemic arterial pressure D. Increase cardiac output
th
(Answer: D/ Reference: Guyton 10 ed, Ch 15, p 157/ MPL 60)

Edema may result from a decrease in:


A. Plasma proteins C. Capillary permeability
B. Interstitial fluid colloid osmotic pressure D. Capillary hydrostatic pressure
(Answer: A/ Reference: Guyton 10th ed, Ch 16, p 169/ MPL 60)

The following contribute to the anatomic dead space:


A. Bronchi C. Alveolar ducts
B. Respiratory bronchioles D. Terminal bronchioles
th
(Answer: A/ Reference: Ganong 20 ed, Ch 34, p 660/ MPL 60)

True of the pulmonary surfactant:


A. Reduces lung compliance
S. DPPC synthesized solely from fatty acids in blood
D. Reduces surface tension more if surface area is smaller
D. Osmophilic lamellated bodies extruded by Type I alveolar cells
(Answer: C/ Reference: Ganong 20th ed, Ch 34, p 657/ MPL 60)

The volume of gas in the lung after a normal expiration:


A. Tidal volume C. Vital capacity
B. Residual volume D. Functional residual capacity
(Answer: D/ Reference: Guyton 10th ed, Ch 37, p 437/ MPL 90)

The diffusing capacity of the lungs is directly proportional to the:


A. Flow of blood C. Amount of gas transferred
B. Thickness of the blood - gas barrier D. Difference in partial pressure
(Answer: D/ Reference: Ganong 20th ed, Ch 34, p 663/ MPL 60)

When arterial pCO2- rises, the main stimulus to increased ventilation comes from the:
A. aortic bodies C. carotid bodies
B. central chemoreceptors D. pulmonary stretch receptors
(Answer: B/ Reference: Guyton 10th ed, Ch 41, p 476/ MPL 60)

Physiology
32 / 50

In a person with chronic obstructive pulmonary disease with chronic retention of CO 2, the main stimulus to ventilation comes from the:
A. J receptors C. carotid & aortic bodies
B. central chemoreceptors D. pulmonary stretch receptors
(Answer: C/ Reference: Guyton 10th ed, Ch 36, p 680/ MPL 60)

The recoil pressures of the lungs and thoracic wall are equal at:
A. end - inspiration C. residual volume
B. mid - inspiration D. functional residual capacity
(Answer: D/ Reference: Guyton 10th ed, Ch / MPL 60)

A decrease in ventilation results from stimulation of:


A. irritant receptors C. proprioceptors of muscles & joints
B. thermal receptors for warmth D. aortic & carotid sinus baroreceptors
(Answer: D/ Reference: Guyton 10th ed, Ch / MPL 60)

Victims trapped in burning buildings suffer from hypoxia because of the formation of:
A. methemoglobin C. carbon dioxide
B. carboxyhemoglobin D. glycosylated hemoglobin
(Answer: B/ Reference: Ganong 20th ed, Ch 37, p 694-5/ MPL 90)

Which of the following is TRUE of Caisson's disease?


A. The gas that is primarily involved is carbon dioxide.
B. The disorder is due to air embolism in the cardiovascular system.
C. The condition occurs as a result of sudden rise in atmospheric pressure.
D. The major cause of death is obstruction of the small respiratory passages.
(Answer: C/ Reference: Ganong 20th ed, Ch 37, p 699/ MPL 60)

The intrapleural pressure is less than intrapulmonary pressure during:


A. quiet inspiration C. both of these
B. quiet expiration D. neither of these
(Answer: A/ Reference: Ganong 20th ed, Ch 34, p 651-2/ MPL 33)

A subject was instructed to exhale maximally. The amount of air present in the lungs after the procedure is known as:
A. residual volume C. vital capacity
B. expiratory reserve volume D. functional residual capacity
(Answer: A/ Reference: Guyton 10th ed, Ch 37, p 437 / MPL 90)

A person breathes in & out into a paper bag. After a few cycles, the alveolar air has:
A. increased pO2 & increased pCO2 C. increased pO2 & decreased pCO2
B. decreased pO2 & increased pCO2 D. decreased pO2 & decreased pCO2
(Answer: B/ Reference: Ganong 20th ed, Ch 37, p 696 / MPL 60)

Which segment of the lung receives the highest perfusion in an upright position?
A. upper third C. medial third
B. middle third D. lower third
(Answer: D/ Reference: Ganong 20th ed, Ch 34, p 660/ MPL 60)

The pivotal reaction in the activation of the complement system involves the activation of complement factor:
A. C1 C. C3
B. C2 D. C4
(Answer: A/ Reference: Guyton 10th ed, Ch 34, p 408/ MPL 90)

Which immunoglobulin class predominates in various bodily secretions?


A. IgA C. IgG
B. IgE D. IgM
(Answer: C/ Reference: Ganong 20th ed, Ch 27, p 530-1/ MPL 90)

Active vaccination often entails more than one dose in order to:
A. produce memory cells.
B. produce longer lasting immunity.
C. deplete the number of effector cells.
D. increase antigen dose to challenge the immune system.
(Answer: A/ Reference: Guyton 10th ed, Ch 27, p 523/ MPL 60)

Which of the following factors/ conditions will shift the oxyhemoglobin dissociation curve to the left?
A. increased pH C. increased pCO2
B. increased 2, 3 DPG D. increased temperature
(Answer: A/ Reference: Ganong 20th ed, Ch 35, p 670-1/ MPL 60)

Which of the following clotting factors is NOT common to both extrinsic & intrinsic pathways of coagulation?
A. Factor I C. Factor III
B. Factor II D. Factor IV
(Answer: C/ Reference: Guyton 10th ed, Ch 36, p 424/ MPL 60)

Physiology
33 / 50

Heparin prevents clotting because it:


A. chelates calcium
B. dissolves fibrinogen
C. blocks the action of thrombin
D. blocks conversion of prothrombin to thrombin
(Answer: C/ Reference: Guyton 10th ed, Ch 36, p 425/ MPL 60)

The sole known physiologic activator of the fibrinolytic system is:


A. Plasmin C. Urokinase
B. Streptokinase D. Prothrombinase
(Answer: C/ Reference: Guyton 10th ed, Ch 36, p 425/ MPL 60)

The Rh blood group system differs from the ABO system in that in the Rh system:
A. The antibodies occur naturally.
B. The agglutinins are of the IgM type.
C. Hemolytic disease of the newborn is less severe.
D. Presence of agglutinins almost always indicates prior sensitization.
(Answer: D/ Reference: Guyton 10th ed, Ch 35, p 415/ MPL 60)

TRUE of the Rh hemolytic disease of the newborn:


A. First pregnancy is the most severely affected.
B. Mother is Rh positive, while fetus is Rh negative.
C. Preformed maternal antibodies cross the placenta to react with fetal Rh+cells.
D. After an erythroblastic child has been born, future children are certainly
erythroblastotic.
(Answer: C/ Reference: Ganong 20th ed, Ch 27, p 541/ MPL 60)

The 1st born infant of a 28-year old Rh-negative woman, who had 2 previous 2 nd trimester abortions, has severe hemolysis & circulatory failure.
Hemolytic disease of the newborn could have been prevented with:
A. Anti-D IgM upon termination of her 1st pregnancy
B. Anti-D IgG during the mother's most recent pregnancy
C. Anti-D IgM during the mother's most recent pregnancy
D. Anti-D IgG upon termination of each of the 1st 2 pregnancies
(Answer: A/ Reference: Ganong 20th ed, Ch 27, p 542/ MPL 90)

Blood typing was done on Mr. X. His serum agglutinated both known A & B cells. The blood type of Mr. X is:
A. A C. AB
B. B D. O
(Answer: D/ Reference: Ganong 20th ed, Ch 27, p 539-40/ MPL 90)

What is the mechanism of autoimmunity in rheumatic fever?


A. The structure of a normal self-antigen is altered.
B. Foreign antigenic substances attach to the cells.
C. Immune complexes attach to the surfaces of cells.
D. A foreign antigen cross-reacts with a normal self-antigen.
(Answer: D/ Reference: Guyton 10th ed, Ch 34, p 410/ MPL 60)

Which hormone binds with receptors found in the cell membrane of target cells?
A. estrogen C. cortisol
B. testosterone D. somatotropin
(Answer: D/ Reference: Guyton 10th ed, Ch 74, p 838-40/ MPL 60)

The most biologically potent of the thyroid hormones:


A. thyroxine C. triiodothyronine
B. thyroglobulin D. monoiodotyrosine
(Answer: C/ Reference: Guyton 10th ed, Ch 76, p 858/ MPL 60)

Thyroid hormone increase the metabolic rate of all body cells EXCEPT:
A. liver cells C. red blood cells
B. myocardial cells D. skeletal muscle cells
(Answer: C/ Reference: Guyton 10th ed, Ch 76, p 862-3/ MPL 60)

The main regulator of normal blood calcium level is:


A. calcitonin C. parathormone
B. cholecalciferol D. anterior pituitary
(Answer: C/ Reference: Guyton 10th ed, Ch 79, p 906/ MPL 60)

Glucagon increases blood glucose level by:


A. promoting glycogenesis. C. promoting glycogenolysis.
B. inhibition of gluconeogenesis. D. promoting fatty acid synthesis.
(Answer: C/ Reference: Guyton 10th ed, Ch 76, p 892/ MPL 60)

The phase of the ovarian cycle when LH reaches a peak level:


A. luteal phase C. ovulatory phase
B. follicular phase D. secretory phase
Physiology
34 / 50

(Answer: B/ Reference: Guyton 10th ed, Ch 81, p 930-32/ MPL 60)

The normal menstrual fluid:


A. is composed purely of fresh blood.
B. usually flows continuously for 2 weeks.
C. may have some necrotic endometrial materials.
D. may clot normally unless flow is in profuse amounts.
(Answer: C/ Reference: Guyton 10th ed, Ch 81, p 937/ MPL 60)

Growth hormone secretion is stimulated by:


A. exercise C. dopamine
B. hyperglycemia D. somatostatin
(Answer: A/ Reference: Ganong 20th ed, Ch 22, p 409/ MPL 60)

One of the following stimulates the production or release of thyroid hormone:


A. thiocyanates C. iodine in high doses
B. propylthiouracil D. iodine in physiologic doses
(Answer: D/ Reference: Guyton 10th ed, Ch 76, p 865/ MPL 60)

After a full meal, the main stimulus for insulin secretion is:
A. lysine C. glucose
B. arginine D. somatostatin
(Answer: C/ Reference: Ganong 20th ed, Ch 19, p 348-9/ MPL 60)

The most potent androgen is:


A. testosterone C. androstenedione
B. dihydrotestosterone D. dehydroepiandrosterone
(Answer: B/ Reference: Guyton 10th ed, Ch 80, p 922/ MPL 60)

One of the following is NOT a physiologic effect of testosterone:


A. increased RBC production
B. skeletal and muscle growth
C. decreased calcium deposition in the bones
D. maturation of internal and external genitalia
(Answer: C/ Reference: Guyton 10th ed, Ch 80, p 924/ MPL 60)

The secretory changes in the 2nd half of the female sexual cycle is a function priarily of this hormone:
A. Estradiol C. Progesterone
B. Luteinizing hormone D. Gonadotropin releasing hormone
(Answer: B/ Reference: Guyton 10th ed, Ch 81, p 936/ MPL 90)

The sexual response of the female differs from that of the male in that:
A. The plateau phase in women is prolonged.
B. The orgasmic phase in women is usually absent.
C. The excitement phase in women is generally shorter & faster.
D. There is usually no refractory period in women during the resolution phase.
(Answer: D/ Reference: Guyton 10th ed, Ch 80-1/ MPL 60)

Secondary peristaltic waves of the esophagus are essentially the same as the primary peristaltic waves. EXCEPT that the former:
A. are faster
B. are initiated by swallowing
C. are not dependent on neural control
D. originate in the esophagus rather than in the pharynx
(Answer: D/ Reference: Guyton 10th ed, Ch 63, p 730/ MPL 33)

Which of the following is TRUE of deglutition?


A. The trachea is closed during the buccal stage.
B. The pharyngeal stage is under voluntary control.
C. During the esophageal stage, respiration is concurrently inhibited.
D. It is a complicated mechanism requiring coordination of many muscle groups.
(Answer: D/ Reference: Guyton 10th ed, Ch 63, p 729-30/ MPL 25)
Which of the following is LEAST likely to inhibit HCl secretion?
A. gastrin C. dietary fat
B. secretin D. prostaglandins
th
(Answer: A/ Reference: Ganong 20 ed, Ch 26, p 896/ MPL 60)

Which of the following is TRUE regarding peristalsis in the small intestine?


A. It is increased after prolonged fasting.
B. It moves chyme towards the pyloric area.
C. It dies out after traveling a short distance.
D. The velocity is faster in the ileum than in the duodenum.
(Answer: C/ Reference: Guyton 10th ed, Ch 63, p 734/ MPL 33)

Physiology
35 / 50

TRUE of the enterogastric reflex:


A. Paralytic ileus that follows distention of the intestines
B. Increased motor activity of the terminal ileum following a meal
C. Inhibition of gastric motility by acidic solutions in the duodenum
D. Mass movement of the colon that follows distension of the stomach
(Answer: C/ Reference: Guyton 10th ed, Ch 63, p 732/ MPL 33)

Cholera toxin stimulates the formation of adenyl cyclase, which promotes:


A. opening of sodium channels
B. opening of potassium channels
C. opening of the chloride channels
D. inactivation of sodium glucose co-transport system
(Answer: C/ Reference: Guyton 10th ed, Ch 65, p 701/ MPL 60)

Which of the following inhibits intestinal motility?


A. Insulin C. Gastrin
B. Glucagon D. Cholecystokinin
(Answer: B/ Reference: Guyton 9th ed, Ch 64/ MPL 60)

One of the following does NOT predispose to gallstone formation:


A. Too much water absorption from the bile
B. Too much lecithin absorption from the bile
C. Too much bile salt absorption from the bile
D. Too much cholesterol absorption from the bile
(Answer: D/ Reference: Guyton 9th ed, Ch 64, p 829/ MPL 60)

If a gallstone were impacted in the distal common bile duct as opposed to the cystic duct, which statement is C0RRECT?
A. The patient would be more likely to have pain.
B. The patient would be more likely to have fever.
C. The patient would be more likely to have jaundice.
D. The patient would be more likely to have nausea & vomiting.
(Answer: C/ Reference: Ganong 20th ed, Ch 26, p 508/ MPL 60)

The following substances decrease the glomerular filtration rate, EXCEPT:


A. endothelin C. norepinephrine
B. epinephrine D. endothelial derived nitric oxide
(Answer: D/ Reference: Guyton 10th ed, Ch 26, p 288/ MPL 90)

The reabsorption of amino acids along the proximal tubules is by:


A. solvent drag C. sodium co-transport
B. facilitated diffusion D. primary active transport
th
(Answer: C/ Reference: Guyton 10 ed, Ch 27, p 300/ MPL 90)

The reabsorption of water is greatest at this segment of the renal tubule:


A. proximal tubule C. ascending loop of Henle
B. distal tubule D. descending loop of Henle
(Answer: A/ Reference: Guyton 10th ed, Ch 27, p 300/ MPL 90)

The absence of albumin in the glomerular filtrate is due to:


A. glomerulo-tubular balance
B. the small diameter of the slit pores
C. markedly increased peritubular oncotic pressure
D. presence of proteoglycan coating the basement membrane
(Answer: D/ Reference: Guyton 10th ed, Ch 26, p 285/ MPL 60)

Profuse vomiting of gastric contentsmay result in this primary acid-base disorder:


A. metabolic alkalosis C. respiratory alkalosis
B. metabolic acidosis D. respiratory acidosis
(Answer: A/ Reference: Guyton 10th ed, Ch 30, p 360/ MPL 90)

Decreased blood flow to the kidneys will result in:


A. vasodilatation C. salt & water secretion
B. aldosterone secretion D. vasopressin secretion
(Answer: B/ Reference: Guyton 10th ed, Ch 26/ MPL 90)

100. Where does aldosterone exert its greatest effect?


A. Glomerulus C. Loop of Henle
B. Proximal tubule D. Cortical collecting duct
(Answer: B/ Reference: Ganong 20th ed, Ch 38, p 719/ MPL 60)

Physiology
36 / 50

MPL
60% _C_ 1. Passage of ions and dipoles through to pores of the cell membrane is affected by the following characteristics of the membrane
permeated –
A. Lipid nature C. Presence of charge
B. Pore size D. Hydrophillic nature

90% _A_ 2. Osmosis –


A. Is downhill transport.
B. Is movement mainly of solute particles through a semipermeable membrane.
C. Is inhibited when metabolic poisons act on the energy system of the cell.
D. Utilizes energy from cellular metabolism.

90% _B_ 3. This organelle in the muscle cell controls the local concentration of Ca++ near the contractile machinery and thereby influences the
contraction relaxation process–
A. Centriole C. Mitochondria
B. Endoplasmic reticulum D. Golgi complex

70% _C__ 4. The ECF is considered the “internal environment” of the body because it is –
A. In equilibrium with the external environment.
B. Constantly changing.
C. Essentially the same for all cells.
D. Made up of 60% of the total body water.

80% _C__ 5. Active transport is characterized by the following; EXCEPT –


A. Utilizes metabolic energy.
B. Utilizes carrier molecules in the cell membrane
C. End-point is uniformity or equal concentrations in the two sides of the membrane.
D. Rate of transport becomes constant as saturation point is reached.

80% _B__ 6. Blood flow to each tissue is almost always precisely controlled in relation to –
A. Cardiac output. C. Blood pressure.
B. Tissue needs. D. Blood volume.

90% _A__ 7. Playing the most significant role in determining blood flow in the circulatory
system is –
A. Radius of blood vessel C. Viscosity of blood.
B. Length of the vessel D. Blood pressure.

80% _D__ 8. The Lymphatic System controls the –


A. Concentration of proteins in the interstitial fluid.
B. Volume of the interstitial fluid.
C. Interstitial fluid pressure.
D. All of the above.

60% _C__ 9. The common denominator in all types of circulatory shock is –


A. Decreased in blood pressure.
B. Decreased in metabolic rate.
C. Diminished tissue perfusion.
D. Decreased in blood volume.

60% _C__ 13. Extracellular edema can be brought about by the following; EXCEPT –
A. Increased capillary pressure.
B. Decreased plasma colloid osmotic pressure.
C. Decrease in interstitial fluid pressure.
D. Lymphatic obstruction.

60% _C__ 14. Because aortic pulse pressure can go as high 120 mmHg and right pulmonary artery pressure averages at 25 mmHg during
systole, the quantity of blood flowing through the lungs is –
A. Less than in the systemic circulation.
B. More than in the systemic circulation.
C. Equal to that in the systemic circulation.
D. More than twice the wedge pressure at recumbent position.

90% _C__ 15. Diffusion and filtration system include the following blood vessels –
A. Arteries C. Capillaries
B. Arterioles D. Veins.

70% _A__ 16. Streamline or laminar flow is characterized by the following –


A. Each layer of fluid slides smoothly over an adjacent layer.
B. Flow velocity increases toward the periphery of the tube.
C. Velocity in inversely proportional to tube radius.
D. Resistance to flow is greatest at the center of the tube.

70% _A__ 17. Low pressure in the pulmonary circulation indicate the following –
A. Dilatable vessels.
Physiology
37 / 50

B. High resistance vessels.


C. Lesser vessel diameter.
D. Less viscous blood.

MULTIPLE CHOICE : Choice the best answer.


MPL
70% __B__ 18. According to Poiseuilli’s law, an increase in the following factor tends to retard fluid flow –
A. Radius of the tube.
B. Viscosity of the fluid.
C. Pressure head.
D. Velocity of flow.

60% _D__ 19. According to the Vasodilator theory of local blood flow regulation, increase in local blood flow can be brought about by –
A. Increased metabolic rate.
B. Diminished oxygen supply.
C. Increased formation of vasodilator substances.
D. All of the above.

60% _C__ 20. Ted’s blood pressure is normally 120/80 mmHg. This morning it rose to 150/100 mmHg. Supposing his kidney are normal, his
renal – body fluid mechanism is expected to respond by –
A. Kidneys will further retain water until a diuretic is given.
B. Blood volume will remain elevated until a vasodilator is given.
C. The rise in blood pressure will cause pressure natriuresis.
D. Increase thirst and increase in water intake.

70% _C_ 21. T’was a crispily cold December morning when Joshua and his Mom and Dad were walking calmly towards the village chapel to
attend the early dawn mass. Breathing in the fresh cool morning air, they were filled with some kind of peace and serenity. As the air is conducted
in the upper respiratory passages, it undergoes the following; EXCEPT –
A. Filtration C. Cooling.
B. Humidification D. Warming.

60% __B__ 22. The ultimate goal of pulmonary ventilation is to –


A. Expand the lungs.
B. Conduct fresh air into the alveoli.
C. Oxidize nutrients for energy production.
D. Aerate the blood.

60% __D__23. Systemic blood supply to the lungs is the bronchial artery which has the following function (s) –
A. To provide nutrition to the airways, larger pulmonary vessels and pleura.
B. To add water vapor to inspired air.
C. To transport oxygenated blood from the alveoli to the right auricle.
D. A and B only.

80% __B__ 24. Fresh inspired air fills up the anatomic dead space which –
A. Extends from the nose and oral cavity until the tertiary bronchi.
B. Is decreased by tracheostomy.
C. Is normally greater than the physiologic dead space.
D. Is always pathological and impairs pulmonary function.

70% _C__ 25. Lung efficiency depends on how closely the following two factors match –
A. Diffusion and transport.
B. Depth of inspiration and expiration.
C. Alveolar ventilation and perfusion.
D. External and internal respiration.

MULTIPLE CHOICE : Choice the best answer.


MPL
60% _C__ 26. If respiratory rate is 15/min. tidal volume = 500 ml and physiologic dead air space = 150 ml, how much is the alveolar ventilation?
A. 4200 ml C. 5250 ml
B. 5000 ml D. 6500 ml

70% _C__ 27. The importance of residual volume is the following –


A. Keeps the lungs inflated at the end of the normal expiration.
B. Makes the lungs fully expanded during inspiration.
C. Provides air in the alveoli to aerate the blood even between breaths.
D. Allows warming of air is alveoli before diffusion into the pulmonary blood.

80% _A__ 28. Normal quiet expiration is entirely a passive process, because –
A. No nerve impulses are sent to the effector muscles.
B. Only inertial work of breathing is performed.
C. No thoracic pressure and volume changes occur.
D. All of the above.

Physiology
38 / 50

90% _C__ 29. This refers to the maximum amount of air a person can expel from the lungs after first filling the lungs to their maximum extent and
then expiring to the maximum extent –
A. Tidal volume C. Vital capacity
B. Inspiratory capacity D. Expiratory reserve volume

80% _B__ 30. The potent effect of CO2 on the neurons in the respiratory center, is due to the following characteristic –
A. High utilization coefficient.
B. Ability to react with carbonic anhydrase.
C. Low molecular weight.
D. High kinetic energy.

60% _A__ 31. Mrs. P. del Rosario was admitted to the hospital for transfusion of whole blood due to a chronic anemia. Her blood type was Type
B, Rh(+) but inadvertently she was given Type A, Rh(+) blood. This transfusion was futile and fatal because –
A. Donor’s RBC with A agglutinogens were destroyed by the patients anti-B angglutinins.
B. Patient’s anti-B agglutinins were destroyed by the donor’s A agglutinogens.
C. Patient’s RBC with A agglutinogens were destroyed by the donor’s anti-B agglutinins
D. Donor’s RBC with A agglutinogens were destroyed by the donor’s B agglutinogens

While walking down a dark alley, one Starless night, Paulo’s right foot accidentally stepped on a piece of broken glass which pierced through his rubber
sandals; He immediately raised his right foot as his arms swayed sidewards as he balanced himself.

70% _A__ 32. The following cord reflex(es) exhibited include the following –
A. Flexor reflex, right foot
B. Cross extensor reflex, right foot
C. Stretch reflex, right foot
D. Golgi tendon reflex, arms

MULTIPLE CHOICE : Choice the best answer.


MPL
60% _D__ 33. A powerful feedback mechanism that begins with tissue inflammation and ends finally with the removed of the cause of
inflammation is provided by the following:
A. Tumor necrosis factor (TNF)
B. Interleukin – 1 (TL-1)
C. Colony stimulating factors (CSF)
D. All

60% _C__ 34. One of the most important of all products of the complement cascade which has a direct effect of rupturing the cell membranes of
bacteria and other invading organisms is:
A. C5a C. C5b6789
B. C3b D. C5b67

70% _A__ 35. The following is / are correct of the intrinsic pathway for initiating clotting:
A. Involves activation of factor XII and release of platelet phospholipids.
B. Initiated by tissue factor such as a traumalized vascular wall.
C. More explosive than the extrinsic pathway.
D. With severe tissue trauma, clotting can occur in as little as 15 secs.

80% _D__ 36. The following is / are correct of agglutinins:


A. Present when the corresponding agglutinogen is absent.
B. Immediately after birth, quantity of agglutinins in the plasma is almost zero.
C. May have a maximum of 10 binding sites thereby causing more RBCs to clump together on a single agglutinin.
D. All of the above.

90% _A__ 37. The most essential regulator of red blood cell production is:
A. Tissue oxygenation
B. Erythropoietin
C. Folic acid and cyanocobalamin
D. Colony-forming unit-erythrocyte (CFU-E)

Case 1 : Anna is almost at the doorstep of her dream of becoming an international flight stewardess. She can almost picture herself elegantly dressed,
flashing her winning grin to all passengers. She has everything calculated well except for her medical exams, particularly the ECG taken a day
before. This could be crucial since she has been experiencing occasional chest pains lately. Can her cardiac status stand the rigorous schedule of
international flights?

80% _A__ 38. In this case, Anna’s ECG tracing can give a clue to the following conditions EXCEPT –
A. Calculate her cardiac output.
B. Detect myocardial ischemia.
C. Show conduction abnormalities.
D. Demonstrate electrolyte alterations.

70% _B__ 39. Her tracing could have erroneous results if –


A. All the 12 conventional leads were recorded.
B. The standardization is not indicated at the start of the tracing.
C. In lead III, the (+) electrode was placed on the left leg and the (-) electrode was placed on the left arm.
D. In V4, the electrode was placed on the 5th ICSLMCL.
Physiology
39 / 50

MULTIPLE CHOICE : Choice the best answer.


MPL
70% _A__ 40. In her ECG, the P wave would tell us about the following:
A. Her atrial depolarization
B. Her atrial repolarization
C. The start of her ventricular depolarization
D. Her P wave is quite insignificant and can be disregarded.

80% _C__41. Her P wave was negative (-) in aVR, what could this mean?
A. There might chamber enlargement of her atria
B. There could be a conduction abnormality from the SA node to the AV node.
C. It is a normal finding.
D. An echocardiogram is indicated to assess the valves and chambers.

60% _D_ 42. A scrutiny of her QRS complex showed that it was normal in voltage and was positive in leads V5 and V6 and was bi-phasic in
leads V3 and V4. This would show:
A. Probable ventricular hypertrophy pattern.
B. Electrolyte imbalance.
C. Ischemic process.
D. It is a normal finding.

70% _D_ 43. Her T wave would show us the following information:
A. If tall and peaked it could indicate hyperkalemia.
B. If inverted in particular leads, it could demonstrate ischemia.
C. Its configuration could be distorted by ventricular hypertrophy.
D. All of the above are true.

70% _ D_ 44. There was no u wave seen in her tracing. If it was seen its significance could
be:
A. Further repolarization of her ventricles.
B. Electrolyte abnormalities particularly hypokalemia.
C. It can be seen also in left ventricular hypertrophy.
D. All of the above are true.

70% _D__ 45. The upright deflection of her QRS in V4-V6 shows that:
A. The depolarization dipole is moving towards the electrode.
B. This is a normal QRS progression.
C. The direction of the movement of the dipole is towards the left.
D. All of the above are true.

Case 2: Six hours prior to admission, Joseph, 46 years old, complained of constricting substernal chest pain while playing basketball with friends. In
contrast to his other episodes of chest pains in the past, this chest pain lasted more than 10 minutes and wasn’t relieved by rest. He at once took
his usual sublingual nitroglycerin tablets, but there was no relief. He was rushed to CDH emergency room and his attending physician’s admitting
diagnosis was myocardial infarct.

60% _A___ 46. If you were Joseph’s cardiologist, which of the following cardiac enzymes
would your request?
A. CK Mb
B. AST
C. LDH
D. All of the above

MULTIPLE CHOICE : Choice the best answer.


MPL
60% _D__ 47. The enzyme levels were not very conclusive. Another cardiac enzyme test was repeated after two days. It was most probably:
A. AST
B. Troponin T/I
C. LDH
D. Any one of the above

60% _D__ 48. The utility and advantage of the Troponin T/I over the other studies could be because:
A. It is more specific when there is concomitant skeletal muscle injury that may confuse results.
B. It could also be more specific when there is a small myocardial infarction that is below the detection limit ok CkMb.
C. It could defect infarct has happened for some days since troponin T could remain elevated for 14 days.
D. All of the above.

70% _D__ 49. On the 4th day post admission, Joseph developed arrhythmia. Its causation could be explained by the following mechanisms:
A. A probable block in the different points of impulse transmission in the heart.
B. There might be spontaneous generation of abnormal impulses in almost any part of the conducting system.
C. Abnormal rhythmicity of the pacemaker.
D. All of the above.

70% __A__ 50. The side-by-side relationship between myosin and action filaments is achieved
by the following –
A. Large numbers of filaments molecules of a protein called titin
Physiology
40 / 50

B. Strong insertion of action filaments into the 2 discs by alpha actinin.


C. Interaction of myosin and actin filaments through the cross-bridges
D. Absence of cross-bridges at the very most center in myosin

70% _D__ 51. A pure actin filament without the presence of the troponin-tropomysin complex binds instantly and strongly with the heads of the
myosin molecules in the presence of the following –
A. Ca ions C. ATP
B. Mg ions D. B and C

60% _B__ 52. Every muscle of the body is composed of a mixture of so-called fast and slow muscle fibers. The differences between these 2
types of fibers include the following –
A. Fast muscle fibers are rich in red myoglobin
B. Fast fibers have less extensive blood supply
C. Slow fibers have fewer mitochondria than fast fibers
D. Slow fibers have large amounts of glycolytic enzymes

60% _C__ 53. Which of the following substances is important for skeletal muscle contraction but not for smooth muscle contraction?
A. Myosin-ATPase C. Troponin
B. Actin D. Ca++-ATPase

MULTIPLE CHOICE : Choice the best answer.


MPL
60% _A__ 54. Which statement below correctly describes the channels located on the end-plate membranes of skeletal muscles?
A. The channels are activated by acetycholine
B. The channels are primarily permeable to Na+
C. When all post synaptic channels are open, the membrane potential becomes positive
D. The number of open channels increases when the membrane depolarizes

60% _B__ 55. On which of the following sensory systems is the transducer and spike generator located on the same cell?
A. Hearing C. Vision
B. Olfaction D. Taste

60% _B__ 56. The following statements are true of the taste sensation EXCEPT –
A. Most taste buds are located around the circumvallate papillae
B. Each taste bud can be excited by more than one of the primary taste stimuli, when the taste substance is in low concentration.
C. The extreme degree of adaptation that occurs in taste sensation occurs in the CNS itself.
D. Phenylthiocarbamide is used frequently to detect taste blindness.

60% _B__ 57. Which type of receptor is activated by Acetylecholine and blocked by atropine?
A. Nicotinic C. Beta adrenergic
B. Muscarinic D. Alpha adrenergic

70% _C__ 58. The association area of the central cortex that provides a high level of interpretive meaning for signals from all surrounding
sensory areas, is the –
A. Prefrontal association area
B. Limbic association area
C. Parieto-occipitotemporal association area
D. Secondary somatic sensory area

60% _D__ 59. In transmission of a neuronal signal from a presynaptic neuron to a postsynaptic neuron, a certain amount of time is consumed
in the following process(es):
A. Discharge of the transmitter substance by the presynaptic terminal
B. Action of the transmitter on the membrane receptor
C. Action of the receptor to increase the membrane permeability
D. All of the above

70% _B__ 60. Which of the following is a feature of both the stretch and the withdrawal
reflexes?
A. After discharge
B. Reciprocal innervation
C. Simultaneous stimulation of alpha and gamma motoneurons
D. Reduction of Ia fiber activity during a contraction

60% _D__ 61. The following events occur when the stapes hits the oval window:
A. Increase in pressure in the scala vestibule
B. Movement of the basilar fibers
C. Excitation of the hair cells
D. All of the above

MULTIPLE CHOICE : Choice the best answer.


MPL
60% _D__ 62. Factors that control gastrointestinal functions are the following, EXCEPT:
A. Enteric Nervous System
B. Autonomic Nervous System
C. Hormones
Physiology
41 / 50

D. No exception

80% _D__ 63. The following conditions make the GI cell membrane more excitable,
EXCEPT:
A. Stretching of the muscle
B. Stimulation by acetylcholine
C. Parasympathetic stimulation
D. Sympathetic stimulation

80% _D__ 64. What factor would like delay gastric emptying:
A. Increase food volume in the stomach
B. Stretching of the stomach wall
C. Gastrin secretion
D. Distention of the duodenum

80% _C__ 65. Mass movements in the colon:


A. Propel the feces from the cecum to the sigmoid
B. Occur 12-24X per day
C. A modified type of persitalsis
D. All of the above

80% _C__ 66. This contributes the greatest amount in the total volume of intestinal juices:
A. Saliva
B. Gastric secretion
C. Small intestine secretions
D. Large intestine secretions

90% _C__ 67. The following statements are true about bile salts, EXCEPT:
A. Important in the emulsification of fats
B. Aid in the absorption of fat soluble vitamins
C. They are not reabsorbed into the blood from the small intestine
D. No exception

80% _C__ 68. This hormone stimulate the secretion of bile by the liver:
A. CCK
B. Histamine
C. Secretin
D. Gastrin

80% _A__ 69. The absorption of glucose by the small intestine is via:
A. Sodium cotransport
B. Diffusion
C. Solvent drag
D. Active transport

90% _C__ 70. These provide the greatest surface area for the absorption in the small intestine
A. Valvulae conniventes
B. Villi
C. Microvilli
D. Taenia coli

MULTIPLE CHOICE : Choice the best answer.


MPL
70% _C__ 71. In athletes on training, the changes that occur inside their hypertrophied
muscles include the following, EXCEPT:
A. Increase in the number of myofibrils.
B. Increase in mitochondrial enzymes.
C. Decrease in stored triglyceride
D. Increase in ATP and phosphocreatine

70% _A__ 72. The final common denominator of cardiovascular function in exercise is to:
A. Deliver oxygen and other nutrients to the muscles.
B. To remove CO2 of other waste products of metabolism from muscles.
C. To increase cardiac output in response to increased venous returns.
D. To strengthen the cardiac muscles to reduce cardiovascular disease.

70% _C__ 73. What metabolic systems in exercise is utilized by the muscle from power surges of a few seconds.
A. Glycogen-lactic system
B. Aerobic system
C. Phosphagen system
D. All of the above

90% _B__ 74. Most substances in the plasma are freely filtered from the glomerulus, EXCEPT:
A. Glucose
B. Proteins
Physiology
42 / 50

C. Sodium
D. Bicarbonates

80% _B__ 75. This characteristic feature is NOT associated with the glomerulus
A. Composed of anastomosing network of capillaries
B. Has a low hydrostatic pressure
C. Capillaries are covered by the epithelial cells
D. Fluid filtered flows into the Bowman’s capsule

70% _B__ 76. The forces that oppose filtration across the glomerular capillaries are the following, EXCEPT:
A. Bowman’s capsule hydrostatic pressure
B. Glomerular hydrostatic pressure
C. Glomerular capillary colloid osmotic pressure
D. Net filtration pressure

70% _C__ 77. Which of these hormones increases the glomerular filtration rate?
A. Epinephrine
B. Norepinephrine
C. Prostaglandin
D. Angiostensin

70% _D__ 78. Which of the following is NOT a major function of autoregulation in the kidneys?
A. Maintain a relatively constant GFR
B. Allow precise control of renal excretion of water
C. Allow control of renal excretion of solutes
D. Maintain a constant blood pressure

MULTIPLE CHOICE : Choice the best answer.


MPL
80% _A__ 79. About 65 percent of the filtered load of sodium and water are reabsorbed by the –
A. Proximal tube
B. Descending loop of Henle
C. Ascending loop of Henle
D. Collecting duct

70% _C__ 80. The part of the loop of Henle that is virtually impermeable to water –
A. Descending thin segment
B. Ascending thin segment
C. Thick ascending segment
D. Tip of Henle

70% _A__ 81. What type of brain waves are found in the EEG of a normal adult person who is awake, quiet and in a resting state of cerebration?
A. Alpha waves
B. Beta waves
C. Theta waves
D. Delta waves

70% _B__ 82. Which of the following is NOT a metabolic effect of growth hormone?
A. Increase rate of protein synthesis
B. Decrease mobilization of fatty acids
C. Decrease rate of glucose utilization
D. Enhance conversion of fatty acids to acetyl-COA

80% _B__ 83. Which of the following features is related to testosterone?


A. Serves to inhibit the descent of testes during the last third month of pregnancy
B. Secretion after puberty causes secondary sexual characteristics to develop
C. Decrease the total quantity of bone matrix
D. Cause calcium retention

70% _A__ 84. The most important temperature receptors for long-term control of body temperature are neurons located in –
A. Preoptic area of the hypothalamus
B. Supraoptic nuclei
C. Postoptic area
D. Paraventricular nuclei

70% _C__ 85. The permeability of the late distal tubule and cortical collecting duct to water is controlled by the concentration of what hormone?
A. Angiotensin
B. Aldosterone
C. Vasopressin
D. Epinephrine

80% _C__ 86. Which of the following substances are NOT reabsorbed by the proximal tubule?
A. Protein
B. Amino acids
C. Urea
Physiology
43 / 50

D. Glucose

MULTIPLE CHOICE : Choice the best answer.


MPL
70% _D__ 87. The kidneys conserve water by excreting a concentrated urine with the basic requirements such as:
A. High level of ADH
B. High osmolality of the renal medullary interstitial fluid
C. High level of urea excretion
D. A and B

70% _D__ 88. Which of the following features is a synaptic function of neurons?
A. An impulse may be locked in its transmission
B. Each impulse may be changed from a single impulse into repetitive impulses
C. Each impulse may be integrated with impulses from other neurons
D. All of the above

70% _A__ 89. What chemical substance is believed to be the major transmitter substance associated with the production of sleep?
A. Serotonin
B. Acetylcholine
C. Dopamine
D. Neorepinephrine

70% _B__ 90. The most important receptors for determining joint angulation in midranges of motion are –
A. Pacinian corpuscles
B. Muscle spindles
C. Golgi tendon apparatus
D. Krause corpuscles

60% _D__ 91. Research work has suggested that this chemical agent might be the single agent most responsible for causing the tissue damage type of
pain:
A. Serotonin
B. Histamine
C. Substance P
D. Bradykinin

70% _B__ 92. The following are physiologic effects of glucocorticoids, EXCEPT:
A. Increase glucose levels in the plasma
B. Decrease protein synthesis and decrease cellular protein catabolism
C. Increase proteins in the plasma and liver
D. Increase levels of blood amino acids

60% _C__ 93. Given the following plasma electrolyte levels, what could possibly be the adrenal disorder:
Na+ - 145mg/dl
K+ - 2.4mg/dl
Cl+ - 96mg/dl
HCO3- - 41
A. Normal
B. Adrenal insufficiency
C. Primary aldosteronism
D. SIADH

MULTIPLE CHOICE : Choice the best answer.


MPL
70% _B__ 94. The following inhibits the production of insulin by the pancreas:
A. Glucose
B. Somatostatin
C. Glucagon
D. Amino acids

60% _D__ 95. Mr. Johns, a 65 y.o. male, recently underwent disarticulation of his right big toe for a non healing ulcer. His physician told him
that he had adult-onset diabetes. What are the effects of insulin deficiency on the tissues that brought about Mr. John’s condition:
A. Inadequate blood supply to the tissues secondary to structural changes of the blood vessel
B. Peripheral neuropathy
C. Increased utilization and decreased storage of proteins
D. All of the above

70% _B__ 96. The rate of iodide trapping by the thyroid cells is influenced by several factors, the most important being the following:
A. Level of T3 in the plasma
B. Concentration of TSH
C. Rate of storage of thyroglobulin
D. Rate of transport of thyroxine and triiodothyronine to the tissues.

80% _C__ 97. Parathyroid hormone maintains normal ionized serum calcium concentration by the following mechanism of action on the bone:
A. Inhibits osteoclastic function
B. Decreases formation of new ostoeclasts
Physiology
44 / 50

C. Increases osteoblast-initiated recruitment of osteocytes and osteoclasts


D. All of the above

60% _D__ 98. The following factors stimulate calcitonin synthesis and secretion, EXCEPT:
A. Cholecystokinin
B. Glucagons
C. -adrenergic agonists
D. -adrenergic antagonists

60% _A__ 99. Which of the following is NOT true of a typical menstrual cycle?
A. Menstruation is induced by an increase in estrogen and progesterone
B. The increase in FSH on day 28 is induced by loss of steroid negative feedback
C. Nondominant follicles undergo atresia due to a decrease in FSH
D. The ratio of LH to FSH increases before ovulation

A 28 year old female underwent thyroidectomy. Four days post-operatively. She had cardio-pulmonary arrest. Her calcium blood level was 6.2 mg/dl.

70% _C__ 100. What could most likely cause her calcium blood level reach this value?
A. She had burst of secretion of her calcitonin prior to the removal of her thyroid gland
B. She has damaged kidneys.
C. Her parathyroid gland was probably accidentally removed.
D. Her bones were unable to provide calcium

Angeles University
COLLEGE OF MEDICINE
PHYSIOLOGY EXAMINATION
December 11, 2003

Name: _________________________________ SCORE: ________________

I. CHOOSE THE CORRECT ANSWER.

_____ 1. The volume of air inspired or expired with each normal breath is called:
A. total lung capacity C. inspiratory capacity
B. tidal volume D. inspiratory reserve volume

_____ 2. A woman has a respiratory rate of 12, a tidal volume of 400 ml, and a dead space air of 100ml. What is her alveolar ventilation?
A. 4.8L C. 3.6 L
B. 4.9 L D. 5.0 L

_____ 3. Normal expiration occurs passively due to:


A. elevation of the ribs
B. downward movement of the diaphragm
C. increase intraabdominal pressure
D. relaxation of inspiratory muscle

_____ 4. During quiet breathing, the intrathoracic pressure is lowest in:


A. early inspiration C. late inspiration
B. mid inspiration D. mid late expiration

_____ 5. A spirometer can be used to measure the following except:


A. tidal volume C. inspiratory reserve volume
B. Vital capacity D. functional residual capacity

_____ 6. Which of the following is correct?


A. TV + IRV = VC C. TV+ ERV + IRV = VC
B. TV + ERV = TLC D. RV + ERV = VC

_____ 7. During inspiration, which of the following is/are correct?


A. the diaphragm contracts
B. volume of thoracic cage decreases
C. intrathoracic pressure increases
D. intrapulmonic pressure increases

_____ 8. What is the respiratory minute volume when DSA is 100ml, RR is 12 and TV 400 ml?
A. 6.0 L C. 4.9 L
B. 5.0 L D. 4.8 L

_____ 9. At what level must the intrapulmonic pressure be for air to rush into
the lungs:
A. lower than atmospheric pressure
B. higher than atmospheric pressure
C. equal to the atmospheric pressure
D. equal to the intrathoracic pressure
Physiology
45 / 50

_____ 10.The exchange of gases in the lungs takes place by which of the
following?
A. facilitated diffusion C. simple diffusion
B. Osmosis D. active transport

_____ 11. The flow of gas across a semipermeable membrane is inversely


proportional to which of the following?
A. surface area C. diffusion coefficient
B. pressure gradient D. thickness of the membrane

_____ 12.The following structures make up the respiratory membrane except:


A. epithelial cells of the alveoli
B. endothelial cells of the capillaries
C. interstitial layer
D. None of the above

_____ 13.Gas with the highest diffusion coefficient:


A. carbon dioxide C. Nitrogen
B. Oxygen D. Helium

_____ 14.The dead space air :


A. occupies the conducting zone
B. does not participate in the exchange of gases
C. is about 1/3 of the tidal volume
D. all of the above

_____ 15.The bulk of carbon dioxide is transported in the blood as:


A. dissolved carbon dioxide C. carbonic acid
B. bicarbonate D. carboxyhemoglobin

_____ 16.Which of the following condition/s will cause the Hemoglobin -


Oxygen dissociation curve to shift to the right?
A. decrease temperature
B. decrease carbon dioxide
C. decrease hydrogen ion concentration
D. decrease pH

_____ 17.Oxygen can be transported in the blood by which of the following?


A. by combining with hemoglobin
B. by dissolving it with the plasma
C. Both
D. Neither

_____ 18.The actively contracting muscle during forcible expiration is:


A. Diaphragm C. external intercostal
B. sternocleidomastoid D. abdominal muscles

_____ 19.Diffusion of gases across the respiratory membrane is decrease by


which of the following?
A. increase in the area of the respiratory membrane
B. increase in the pressure gradient
C. increase in temperature
D. increase in the molecular weight of diffusing substance

_____ 20.The main factor that will cause diffusion of oxygen across the
respiratory membrane is:
A. pressure gradient C. temperature
B. diffusion coefficient D. None of the above

_____ 21.Rapid transmission of impulses from exterior of the cell to the inside is made possible by:
A. longitudinal tubules C. terminal cisternae
B. transverse tubules D. sarcosome

_____ 22.The dark bands are composed of:


A. actin filaments C. myosin filaments
B. actin and myosin filaments D. Z-lines

_____ 23.The light bands:


A. contain actin & myosin filaments
B. are isotropic to polarized light
C. get narrower during contraction
D. comprise the length of the thick filament

Physiology
46 / 50

_____ 24.The sarcomere:


A. includes the entire I band and half of A band on each side of I
B. is a portion of a myofibril that lies between 2 t-tubules
C. reveals the H-zone in a fully stretched muscle fiber
D. when fully contracted, shows barely overlapping actin & myosin filaments

_____ 25.Calcium in the sarcoplasmic reticulum is principally stored by:


A. passive calcium pumps C. active calcium pumps
B. calsequestrin D. calmodulin

_____ 26.Ions surrounding the sarcolemma give the muscle the ability to propagate action potential. This physiologic property is:
A. excitability C. elasticity
B. contractility D. conductivity

_____ 27.ATP is needed in the following situations, EXCEPT:


A. to energize the powerstroke of the crossbridge
B. to reposition the crossbridge
C. to trigger the exposure of actin binding sites
D. to transport the calcium back to to the sarcoplasmic reticulum

_____ 28.The multi-unit smooth muscle is seen in:


A. bile ducts C. piloerector muscle
B. capillaries D. ureter

_____ 29.Smooth muscle fiber contains the following, EXCEPT:


A. actin C. tropomyosin
B. troponin C D. calmodulin

_____ 30.The following statements are true regarding smooth muscle, EXCEPT:
A. smooth muscles are slow to contract and relax because of fewer myosin
molecules
B. calcium comes from the ECF and not the sarcoplasmic reticulum
C. the actin filaments are attached to dense bodies instead of Z discs
D. Ca++ pumps are slow-acting during relaxation

_____32. REM sleep is characterized by:


A. slow synchronous waves on EEG
B. steady vital signs
C. presence of dreams
D. presence of delta waves

_____33. A sleep disorder characterized by an irresistible urge to sleep during daytime:


A. narcolepsy C. somnambulism
B. insomnia D. bruxisms

_____34. The waves of EEG that are present during REM sleep:
A. alpha waves C. delta waves
B. beta waves D. theta waves

_____ 35. The resting membrane potential of large myelinated nerve fibers is:
A. +90 mV C. -90 mV
B. -65 mV D. + 65 mV

_____ 36.The spike of the action potential is caused by:


A. Sodium ion influx C. Sodium ion efflux
B. Potassium ion influx D. Potassium ion efflux

_____ 37.The potential level that will stop the net diffusion of an ion under appropriate conditions is the:
A. threshold potential C. Goldmann potential
B. Nerst potential D. membrane potential

_____ 38. All are type C fibers except :


A. pain fibers C. annulospiral endings
B. itch fibers D. Crude tower fibers

_____ 39.The following are chemoreceptor functions, EXCEPT :


A. detection of oxygen level in the arterial blood
B. detection of light on the retina of the eye
C. detection of osmolality of the body fluids
D. detection of odor in the nose

_____ 40.Gray layers of the sensory cerebral cortex include:


A. pyramidal cells of Beth C. external granular cells
B. polymorphonuclear cells D. all of the above

Physiology
47 / 50

_____ 41.Somatic area I contains innervation from:


A. thigh C. tongue
B. fingers D. all of the above

_____ 42.Presynaptic terminals coming from other neurons mainly attach to this
part of the target neuron
A. Dendrites C. Axon
B. Soma or body D. None of the previous

_____43. An inhibitory neurotransmitter will activate this ion channel at the


postsynaptic membrane
A. Sodium channel C. Chloride channel
B. Calcium channel D. None of the previous

_____44. All of the following are inhibitory neurotransmitters EXCEPT :


A. Dopamine C. Serotonine
B. Histamine D. Glycine

_____45. The excitatory postsynaptic potential needed to bring a resting cell to the
threshold for firing is:
A. +10mV C. +20mV
B. +15mV D. +25mV

_____46. This factor affects synaptic transmission by increasing neuronal excitability


A. Hypoxia C. Acidosis
B. Strychnine D. Alkalosis

_____47. Process of integration of functions in a highly developed brain


A. sacralization C. encephalization
B. caudalization D. intellectualization

_____48. That part of the CNS that commands and executes willed movements:
A. cerebellar cortex C. thalamus
B. cerebrum D. spinal cord

_____49. That part of the brainstem which serves as relay for visual and auditory
impulses
A. medulla oblongata C. midbrain
B. pons D. basal ganglia

_____50. True regarding the motor homonculus


A. the perineal area has greater representation than the hand
B. cortical motor area controls musculature on the ipsilateral side of the body
C. increased blood flow correlates with increased neuronal activity
D. the entire face is bilaterally represented

_____51. The most important output pathway in initiating discrete voluntary


movement from the motor cortex
A. cortico-ponto-cerebellar tract
B. pyramidal tract
C. cortico-rubral tract
D. cortico-cerebellar tract

_____52. Inhibition of surrounding neurons upon stimulation of an anterior motor


neuron is a specific function of:
A. anterior motor neuron C. propiospinal fibers
B. gamma motor neuron D. Renshaw cells

_____53. Striking the patellar tendon with neurohammer will help determine
sensitvity to this reflex:
A. stretch C crossed extensor
B. flexor D. nociceptive

_____54. Oldest lobe of the cerebellum and functions to control equilibrium and
postural movements:
A. vermis C. olive
A. flocculonodular lobe D. posterior lobe

_____55. Rapid movements of the body such as in typing, occur so rapidly that it is
not possible to receive feedback information either from the periphery to the cerebellum back to the motor cortex before the
movements are over. This is called :
A. timing C. sequential
B. ballistic D. extra-predictive

Physiology
48 / 50

_____56. When the receptor portion of the muscle spindle is stretched slowly, the
number of impulses increases directly in proportion to the degree of
stretching. This phenomenon is known as:
A. dynamic response C. muscle stretch reflex
B. static response D. damping mechanism

_____57. The following excite the muscles of the vertebral column and extensors of
the lower limbs to provide stability in standing against gravity, EXCEPT :
A. medullary reticular nuclei C. fastigial nuclei
B. pontine reticular nuclei D. vestibular nuclei

_____58. Not a part of the equilibrium apparatus needed for detecting head
orientation with gravity
A. semicircular ducts C. utricle
B. cochlea D. saccule

_____59. Transection at the superior border of the pons leads to accentuation of


reflexes due to :
A. removal of facilitatory control by higher centers
B. loss of differentiation of reaction
C. innervation hypersensitivity
D. denervation hypersensitivity above the transection

_____60. TSH acts on the thyroid follicles by:


A. direct interaction with the DNA
B. activation of adenylate cyclase
C. autophosphorylation of tyrosine kinase by the receptor
D. activation of protein channel

_____61. Which of the following hormones directly activate gene transcription?


A. cortisol C. LH
B. prolactin D. calcitonin

_____62. The following statement/s is/are TRUE about CRH?


A. The peak of secretion is in the morning
B. It activates the corticotrophs of the pituitary
C. It’s secretion is further elevated by stress
D. All of the above are true
E. Only A and B are true

_____63. Which of the following hormones is secreted maximally before waking up:
A. growth hormone C. prolactin
B. somatostatin D. cortisol

_____64. Oxytocin will produce which of the following effects?


A. contraction of the smooth muscle of blood vessels
B. stimulation of milk protein gene
C. contraction of the myoepithelial cells of the breast
D. all of the above

_____65 Which of the following anterior pituitary hormones depend on the beta subunit for its biologic specificity?
A. aldosterone C. ACTH
B. corticotropin D. TRH

_____66. Which of the following hypothalamic factors decreases the secretion of prolactin?
A. VIP C. Dopamine
B. TRH D. serotonin

_____67. Inhibition of growth hormone secretion occurs when


A. a person is under physical stress C. the blood amino acids are elevated
B. the blood sugar is elevated D. all of the above

_____68. Which of the following may lead to dwarfism?


A. decreased GHRH pulses from the hypothalamus
B. decreased IGF-1 levels
C. decreased thyroid hormone while in utero and before puberty
D. all of the above
E. A and B only

_____69. TSH will increase which of the following in the thyroid gland?
A. iodide pump activity D. all of the above
B. follicular cell size and number E. A and B only
C. blood supply to the gland

Physiology
49 / 50

_____70. Thyroid hormones will increase all of the following EXCEPT;


A. oxygen consumption D. cardiac contractility and rate
B. bone turnover E. body weight
C. motility of the GIT

_____71.Which of the following statement/s is/are TRUE about cortisol?


A. It decreases migration of neutrophils to sites of tissue injury
B. It stimulates Antibody production
C. It delays fetal lung maturity
D. All of the above

_____72. Insulin will increase all of the following processes EXCEPT:


A. hepatic glycogenesis C. hepatic gluconeogenesis
B. adipose cell lipogenesis D. glucose uptake

_____73. Which of the following stimuli will increase the secretion of insulin by the beta cells
in response to a meal?
A. TRH C. GIP
B. GABA D. VIP

_____74. All of the following processes are increased by glucagon EXCEPT


A. glycogenolysis C. lipolysis
B. gluconeogenesis D. amino acid uptake

_____75. Which of the following comprise natural immunity?


A. intact skin and mucous membranes D. macrophage derived cytokines
B. phagocytes E. all of the above
C. complement

_____76. Which of the following is a part of the specific immunity?


A. gamma interferon C. tumor necrosis factor
B. macrophage D. none of the above

_____77. The immunity passed on from the mother to the fetus through the placenta is
A. active, humoral immunity C. passive, cellular immunity
B. passive, humoral immunity D. active, cellular immunity
_____78. Which of the following immunoglobulins is involved in the trans-placental transfer
of immunity?
A. IgA C. IgD
B. IgG D. IgE

_____79. The immune response to an encounter with a foreign antigen that has been
encountered before by the body is called
A. primary response that has high Ig titers
B. secondary response that has high Ig titers
C. primary response that has low Ig titers
D. secondary response that has low Ig titers

_____80. All of the following characterize specific immunity EXCEPT :


A. self limitation D. diversity
B. discrimination of self from non self E. memory
C. non specific

_____81. Which of the following explains why our body does not produce antibodies against
self antigens?
A. the self antigens are bound to MHC class II molecules
B. the B cells remove the self antigens
C. the T cells against self antigens are negatively selected and die
D. all of the above are true

_____82. The specificity of the immune response is brought about by the interaction of
A. the epitope of the antigen C. both of the above
B. the hypervariable region of the antibody D. neither of the above
_____83. Immunologic memory is possible because
A. Of the development of development of memory B cells
B. each exposure to the antigen increases the clones of lymphocytes specific for that antigen
C. both of the above
D. neither of the above

_____84. The immune response is self limiting because


A. elimination of the antigen removes the stimulus for the immune response
B. lymphocytes perform functions only for brief periods and become quiescent
C. of antibody feedback
D. all of the above
E. A and B only
Physiology
50 / 50

_____85. The ammonia synthesized from the kidney comes mainly from
A. glycine C. glutamine
B. alanine D. tryptophan

_____86. Patient Y was admitted to the hospital with a PaCO2 of 40mmHg, HCO3 of 6 meq/l
and a pH of 7.1. Patient Y has which of the following acid base disorders
A. metabolic acidosis C. respiratory acidosis
B. metabolic alkalosis D. respiratory alkalosis

_____87. The secretion of H+ in the proximal tubule is primarily accompanied by


A. excretion of Na+ C. reabsorption of HCO3
B. reabsorption of phosphate D. secretion of K+

_____88. This statement regarding rennin is correct.


A. it is synthesized in the proximal tubule
B. it converts angiotensin I to angiotensin II
C. it is primarily stimulated by increased ABP
D. it’s secretion leads to Na| and water retention

_____89. Which of the following statements regarding the renal acid-base regulation is correct?
A. excreting an acidic urine increases the acid in the ECF
B. excreting a basic urine removes acids from the ECF
C. kidneys control acid balance by excreting only acidic urine
D. reabsorbing HCO3- will increase the base in the ECF

_____90. Hydrogen ion secretion and bicarbonate reabsorption occurs in all parts of the
tubule except
A. proximal convoluted tubule C. distal tubule
B. collecting duct D. loop of Henle

_____91. The kidneys have an endocrine function because of the secretion of


A. Vasopressin C. angiotensin
B. Aldosterone D. erythropoietin

_____92. Which of the following systems readily combine with acid or base to prevent acidosis or alkalosis?
A. chemical acid – base system C. renal system
B. respiratory system D. gastrointestinal system

_____93. The most important extracellular buffer is


A. bicarbonate buffer system C. ammonia buffer system
B. phosphate buffer system D. hemoglobin buffer system

_____94. Metabolic acidosis can be due to


A. diabetes mellitus C. excess aldosterone
B. vomiting (gastric contents) D. ascent to high altitude area

_____95. In metabolic acidosis, which of the following would be greater than normal
A. pH C. PCO2
B. hydrogen ion concentration D. HCO3

_____96. Site of activation of angiotensin I to angiotensin II


A. kidney C. lungs
B. liver D. adrenal cortex

_____97. The filtered HCO3 within the proximal tubular lumen is reabsorbed mainly as
A. CO2 C. HCO3
B. H2CO3 D. OH-

_____98. In metabolic alkalosis, the primary disturbance is


A. CO2 excess C. HCO3 excess
B. H2CO3 excess D. H+ excess

_____99. In patients with metabolic acidosis, which of the following can be observed
A. hypomentation C. excess CO2
B. increased pH D. decreased HCO3

_____100. Enhances the reabsorption of water in the distal tubule and the collecting duct
A. ADH C. Angiotensin II
B. Aldosterone D. Parathormone

Physiology

You might also like